Download as pdf or txt
Download as pdf or txt
You are on page 1of 42

Page 1 of 42

PTS 2021|Simulator Test 3– Solutions|ForumIAS


Q.1)
Ans) d
Exp) Option d is correct.
Statement 1 is incorrect. According to Article 243B, three-tier panchayat system, at the village, intermediate
and district level, shall be created only in a state having a population above twenty lakhs. In a state having a
population below 20 lakhs panchayats at the intermediate level may not be constituted.
Statement 2 is incorrect. Article 243H provides that the legislature of a state may, by law, authorise a
panchayat to levy, collect and appropriate such taxes, duties, tolls and fees. The state legislature may also
assign to a panchayat such taxes, duties, tolls and fees levied and collected by the State Government.
Source: Indian Polity by M. Laxmikanth. Chapter: Panchayati Raj
Sub) Polity, Indian Constitution, Panchayati Raj - functions vs Parastatal institutions

Q.2)
Ans) b
Exp) Option b is correct.
Statement 1 is incorrect. Grama was the lowest unit of administration which was under gramika. Satavahana
kingdom was divided into subdivisions called aharas or rashtras, meaning districts. Ahara was divided into
Grama.
Statement 2 is correct. Gaulmika was the head of military regiment consisting of nine chariots, nine
elephants, 25 horses and 45 foot soldiers. He was the administrator in the rural areas.
Statement 3 is correct. Katakas and Skandhavaras were military camps and settlements which served as
administrative centres as long as king was present there. Knowledge Base: Satavahanas became prominent
in the Indian political scene sometime in the middle of the first century BC. Gautamiputra Satakarni (first
century AD) is considered to be the greatest of the Satavahana rulers. He is credited with the extension of
Satavahana dominions by defeating Nahapana, the Shaka ruler of Western India. His kingdom is said to
have extended from river Krishna in south to river Godavari in north. The Satavahanas had their capital at
Pratishthana (modern Paithan) near Aurangabad in Maharashtra.
Source: Ancient History, NCERT XI, Chapter-16, Pg. 117-118
NIOS, Module 1 Ancient India, Chapter 6.
Sub) History, Ancient History, satvahana administration

Q.3)
Ans) b
Exp) Option b is correct.
Option a is correct: Government as well as private sector employees are allowed to make voluntary
contributions over and above the statutory deductions into the general provident fund (GPF) or EPF,
respectively.
Option b is incorrect: The budget for FY2021-22 proposed that the interest earned on the EPF contributions
(only employee contribution) above ₹2.5 lakh a year will now be taxable.
Option c is correct: The government had capped the contributions by employers into employee welfare
schemes like the EPF or the National Pension Scheme or a superannuation plan, at ₹7.5 lakh a year, in last
year’s Budget.
Option d is correct: EPF interest rate is declared every year by the EPFO. Recently, the Central Board of
Trustees of the Employees Provident Fund Organisation (EPFO) recommended that subscribers be given
8.5% interest rate for PF contributions for 2020-2021. The rate of 8.5% remains the lowest since 2012-2013.

ForumIAS Offline Guidance Centre


2nd Floor, IAPL House, 19, Pusa Road, Karol Bagh, New Delhi – 110005 | helpdesk@forumias.academy|9821711605
Page 2 of 42

PTS 2021|Simulator Test 3– Solutions|ForumIAS


Source: https://www.thehindu.com/news/national/123-lakh-high-net-worth-individuals-deposited-
62500-crore-into-epf-accounts-in-2018-19/article33753096.ece
https://www.thehindu.com/business/Economy/epfo-fixes-85-interest-on-epf-deposits-for-2020-
21/article33987718.ece
Sub) Economics, Schemes, Employees’ Provident Funds (EPF) Scheme - budget changes

Q.4)
Ans) a
Exp) Option a is correct.
Option a is correct. Dehing Patkai has been upgraded to National Park from Wildlife Sanctuary recently to
protect the sanctity of area from coal and oil mining exploration. It is known as Jeypore rainforest having
fauna like Malayan sun bear and marbled cat. It is also referred as The Amazon of East and is home to
Asiatic Elephant.
Option b is incorrect. Dibru-Saikhowa is a National Park as well as a Biosphere Reserve situated in the south
bank of the river Brahmaputra in the extreme east of Assam state in India.
Option c is incorrect. Nameri National Park is located in the foothills of the Eastern Himalayas in the
Sonitpur District of Assam. The area is criss-crossed by the river Jia- Bhoroli and its tributaries namely the
Diji, Dinai, Doigurung and Nameri.
Option d is incorrect. Orang National park is the oldest game reserve of the Assam on the northern bank of
river Brahmaputra in the Darrang and Sonitpur districts. It is also known as the mini Kaziranga National
Park.
Source: https://economictimes.indiatimes.com/news/politics-and-nation/assam-to-upgrade-dehing-
patkai-wildlife-sanctuary-into-national-park/articleshow/76820603.cms
https://indianexpress.com/article/north-east-india/assam/assam-govt-to-upgrade-dehing-patkai-wildlife-
sanctuary-to-national-park-6493214/
https://www.pratidintime.com/dehing-patkai-upgraded-to-national-park/
https://www.kaziranga-national-park.com/dibru-saikhowa-national-park.shtml
Sub) Geography, Indian Physical Geography, National Park in News (Try to ask that has not been asked in
UPSC)

Q.5)
Ans) c
Exp) Option c is correct.
The Delimitation commission is established by the Government of India under the provisions of the
Delimitation Commission Act. The function of the commission is to set or redraw the boundaries of the
various assembly and Lok Sabha constituencies based on the recent census.
The Commission is a powerful and independent body whose orders cannot be challenged in any court of
law. The orders are laid before the Lok Sabha and the respective State Legislative Assemblies. However,
modifications are not permitted.
Under Article 82, the Parliament enacts a Delimitation Act after every Census.
Source: https://indianexpress.com/article/explained/delimitation-commission-northeast-india-election-
commission-6510333/
Sub) Polity, Indian Constitution, Delimitation Commission-Definition

ForumIAS Offline Guidance Centre


2nd Floor, IAPL House, 19, Pusa Road, Karol Bagh, New Delhi – 110005 | helpdesk@forumias.academy|9821711605
Page 3 of 42

PTS 2021|Simulator Test 3– Solutions|ForumIAS


Q.6)
Ans) d
Exp) Option d is correct.
Relevance: United Nations’ FAO names 4 Asian tea cultivation sites as Globally Important Agri Heritage
Systems
Statement 1 is incorrect: The Food and Agriculture Organization of the United Nations (FAO) leads the
GIAHS Programme, which is intended for the identification and enhancement of the benefits of these
dynamic systems to mitigate the threats they face.
Statement 2 is incorrect: In our country so far, the following sites have received recognition under
this programme: Traditional Agricultural System, Koraput, Odisha, Below Sea Level Farming
System, Kuttanad, Kerala, Saffron heritage of Kashmir in Pampore region.
Source: https://www.financialexpress.com/lifestyle/united-nations-fao-names-4-asian-tea-cultivation-
sites-as-globally-important-agri-heritage-systems/2160865/
Sub) Environment, Convention / Organisation/ Laws/ Protocols, Globally Important Agriculture Heritage
Systems

Q.7)
Ans) d
Exp) Option d is correct.
The power to summon and dissolve the House of the State Legislature is not a discretionary power of the
governor.
The Supreme Court, in 2016 Arunachal Pradesh case, has ruled that, in ordinary circumstances during the
period when the Chief Minister and his council of ministers enjoy the confidence of the majority of the
House, the power vested with the Governor under Article 174 to summon, prorogue and dissolve the
house(s) must be exercised in consonance with the aid and advice of the chief minister and his council of
ministers. In the above situation, he is precluded [from taking] an individual call on the issue at his own will,
or in his own discretion.
The governor has constitutional discretion in the following cases:
1) Reservation of a bill for the consideration of the President.
2) Recommendation for the imposition of the President’s Rule in the state.
3) While exercising his functions as the administrator of an adjoining union territory (in case of additional
charge).
4) Determining the amount payable by the Government of Assam, Meghalaya, Tripura and Mizoram to an
autonomous Tribal District Council as royalty accruing from licenses for mineral exploration.
5) Seeking information from the chief minister with regard to the administrative and legislative matters of
the state.
The governor, like the president, also has situational discretion (i.e., the hidden discretion derived from the
exigencies of a prevailing political situation) in the following cases:
1) Appointment of chief minister when no party has a clear-cut majority in the state legislative assembly or
when the chief minister in office dies suddenly and there is no obvious successor.
2) Dismissal of the council of ministers when it cannot prove the confidence of the state legislative
assembly.
3) Dissolution of the state legislative assembly if the council of ministers has lost its majority.
Source: Indian Polity by M. Laxmikanth. Chapter 26. The Governor
Sub) Polity, Indian Constitution, Power of Governor

ForumIAS Offline Guidance Centre


2nd Floor, IAPL House, 19, Pusa Road, Karol Bagh, New Delhi – 110005 | helpdesk@forumias.academy|9821711605
Page 4 of 42

PTS 2021|Simulator Test 3– Solutions|ForumIAS


Q.8)
Ans) d
Exp) Option d is correct.
Statement 1 is incorrect. Article 142 (1) provides that the Supreme Court in the exercise of its jurisdiction
may pass such decree or make such order as is necessary for doing complete justice in any cause or matter
pending before it. Only the Supreme court has the power.
Article 142 has been used in multiple times. For example, in 2016 the supreme court banned the sale of
alcohol within a distance of 500 meters on national and state highways across the country.
Statement 2 is incorrect. Article 142(2) provides that, the Supreme Court shall have all and every power to
make any order for the purpose of securing the attendance of any person, the discovery or production of any
documents, or the investigation or punishment of any contempt of itself.
However, this power is limited by and subjected to any law made by the parliament.
Source: The Constitution of India by P.M. Bakshi. Page 171
Sub) Polity, Indian Constitution, Article 142

Q.9)
Ans) c
Exp) Option c is correct.
The Global Terrorism Index is annually published by the Institute for Economics & Peace (IEP). IEP is an
independent, non-partisan, non-profit think tank dedicated to shifting the world’s focus to peace as a
positive, achievable, and tangible measure of human well-being and progress. IEP is headquartered in
Sydney, Australia and it works with a wide range of partners internationally and collaborates with
intergovernmental organisations on measuring and communicating the economic value of peace.
India was ranked at 8th position, with the score of 7.353, in Global Terrorism Index 2020. India has the
lowest number of terrorism deaths among the ten countries most affected by terrorism, and the lowest
lethality rate of attacks. But compared to other countries amongst the ten most impacted, India faces a wider
range of terrorist groups, with Islamist, communist, and separatist groups active across the country.
Source: https://niti.gov.in/decoding-global-terrorism-index
Sub) International Relations, Reports & Indices, Global Terrorism Index

Q.10)
Ans) a
Exp) Option a is correct
Statement 1 is correct. Bharat QR code is an interoperable payment acceptance solution that supports Visa,
MasterCard. Amex and RuPay cards & BHIM-UPI for wider acceptance.
Statement 2 is incorrect. Bharat QR code does not require any upfront expenditure as it uses a printed QR
code and customer uses mobile phone to make the payment.
Statement 3 is incorrect. Bharat QR code has been developed by National Payments Corporation of India,
Master card and Visa. It has been devised based on the directions set up by Reserve Bank of India.
Knowledge Base: To facilitate massive rollout in a short span of time, Bharat QR code-based payment
solution is introduced with following advantages:
1) Customer can easily make payments through Bharat QR code and does not require to carry physical Debit
or Credit card.
2) The risk of data theft or security issues through tampered or cyber-compromised point of sale devices is
minimised.

ForumIAS Offline Guidance Centre


2nd Floor, IAPL House, 19, Pusa Road, Karol Bagh, New Delhi – 110005 | helpdesk@forumias.academy|9821711605
Page 5 of 42

PTS 2021|Simulator Test 3– Solutions|ForumIAS


3) Bharat QR code supports dynamic QR codes, which may be printed on electricity bills, gas bills and other
utility bills to make payments to the respective vendors.
4) Merchants accepting the payment through Bharat QR code, receives the amount directly in their Bank
accounts.
Source: https://www.meity.gov.in/bharat-qr-
code#:~:text=Bharat%20QR%20code%20is%20a,physical%20Debit%20or%20Credit%20card.
https://vikaspedia.in/e-governance/digital-payment/cards-for-digital-payments/bharat-
qr#:~:text=Bharat%20QR%20is%20the%20world,)%2C%20Master%20card%20and%20Visa
Sub) Science and Technology, IT, BarCode, Bharat QR code

Q.11)
Ans) b
Exp) Option b is correct.
Statement 1 is incorrect: The 2021 Rules add that CSR funds may be spent for creation or acquisition of
capital asset, which can be held by: (a) a trust or society with a CSR registration number, (b) beneficiaries of
the CSR project, (c) public authority.
Statement 2 is correct: A company can undertake CSR: (a) by itself, or in conjunction with a trust or society
established by the company, or (b) through an entity, trust or society established by the central or state
government, or (c) through a registered trust or society with a track record of at least three years of similar
activities. The latest rules require every entity undertaking CSR activities to register itself with the central
government with effect from April 1, 2021.
Statement 3 is incorrect: Companies whose CSR obligation is greater than ten crore rupees will have to
prepare an impact assessment report for all CSR projects where expenditure is greater than one crore
rupees.
Source: https://prsindia.org/policy/monthly-policy-review/january-2021
Sub) Economics, Indian Economy, Corporate social responsibility - decreased due to covid+ change.

Q.12)
Ans) d
Exp) Option d is correct.
Statement 1 is correct. The Natya Shastra of Bharat Muni mentions many regional varieties, such as the
south-eastern style known as the Odhra Magadha which can be identified as the earliest precursor of
present day Odissi. It was primarily practised by the ‘maharis’ or the temple dancers and patronised by the
Jain king Kheravela.
Statement 2 is correct. The tribhanga posture, i.e. the three-bended form of the body is innate to Odissi
dance form. Also, the ‘Chowk’ posture with hands spread out depicts masculinity. Odissi dance form is
unique in its representation of gracefulness, sensuality and beauty. The dancers create intricate geometrical
shapes and patterns with her body. Hence, it is known as ‘mobile sculpture’.
Statement 3 is correct. The opening item is Mangalacharan where the dancer slowly enters the stage with
flowers in her hands and makes an offering to mother earth. This is followed by an invocation to the deity
of the dancer's choice. The concluding item of the dance is called moksha.
Knowledge Base: http://ccrtindia.gov.in/odissi.php
Source: The theme of Odissi dance has been derived from the Ashtapadis of Jayadeva’s Gita Govinda.
Sub) Art and Culture, Dance, Odissi

ForumIAS Offline Guidance Centre


2nd Floor, IAPL House, 19, Pusa Road, Karol Bagh, New Delhi – 110005 | helpdesk@forumias.academy|9821711605
Page 6 of 42

PTS 2021|Simulator Test 3– Solutions|ForumIAS


Q.13)
Ans) c
Exp) Option c is correct.
Relevance: https://www.thehindu.com/opinion/lead/a-case-for-down-to-earth-
governance/article32555376.ece
https://www.downtoearth.org.in/news/governance/fifteenth-finance-commission-over-rs-4-36-lakh-
crore-for-local-governments-75296
Statement 1 is incorrect. In 1882 Lord Ripon divided all sources of revenue into three groups- General or
Imperial (going entirely to centre), Provincial (going entirely to provinces) and Divided (between centre and
provinces).
Statement 2 is correct. Government of India act 1919, separated for the first time the provincial and central
budgets, with provincial legislatures being authorised to make their budgets.
Statement 3 is correct. The Government of India,1935 Act allowed for the sharing of Centre’s revenues and
for the provision of grants-in-aid to provinces.
Source: Modern History, Spectrum, 6th Edition, Chapter-26, Pg. 528-531
https://www.rbi.org.in/Scripts/PublicationsView.aspx?id=19340
Sub) History, Modern History, Financial decentralization

Q.14)
Ans) d
Exp) Option d is correct.
Statement 1 is correct: plastics exposed to sea water tends to concentrate toxic and non-toxic organic
compounds present in the sea water at low concentrations. These, including PCBs, DDT, and
nonylphenols, have very high partition coefficients and are very efficiently concentrated in the plastic
material.
Statement 2 is correct: The United Nations Environment Programme (UNEP) defines microplastics as
plastic particles smaller than 5 millimetre. Microplastics are intentionally added in primary sources such as
personal care products and clothing products; or secondary sources that are formed by fragmentation of
plastic over a period of time.
Statement 3 is correct: It was estimated that global production of plastics is approximately 250mt/yr. Their
abundance has been found to transport persistent organic pollutants, also known as POPs. These pollutants
have been linked to an increased distribution of algae associated with red tides.
Source: https://www.ncbi.nlm.nih.gov/pmc/articles/PMC2873009/
Relevance: The pandemic has seen a surge in single-use plastics, with disposable masks, gloves and other
PPE equipment ceaselessly washing up on beaches across the globe.
Sub) Environment, Pollution, Plastics Pollution

Q.15)
Ans) d
Exp) Option d is correct.
Statement 1 is correct: Gokhale was loved and recognised by the people. Proud of his legislative achievement
they were to confer him the title of ‘the leader of the opposition’.
Statement 2 is correct: He was a protégé of Ranade and influenced by the British philosopher-
parliamentarian Edmund Burke. Gokhale worked towards realising constitutional ideals in India for three
decades and abjured the use of reactionary or revolutionary ways.

ForumIAS Offline Guidance Centre


2nd Floor, IAPL House, 19, Pusa Road, Karol Bagh, New Delhi – 110005 | helpdesk@forumias.academy|9821711605
Page 7 of 42

PTS 2021|Simulator Test 3– Solutions|ForumIAS


Statement 3 is correct: He joined as a professor at the Fergusson College in Pune, where he taught political
economy and history. Gokhale hailed from the Ratnagiri district, in present-day Maharashtra and studied at
the Elphinstone College in Mumbai.
Knowledge Base: Gokhale first arrived on the national scene after cross-examining British colonial
expenditure at the Welby Commission of 1897 in England. Gokhale’s work earned him praise in India as he
laid bare British military financing policies that heavily burdened Indian taxpayers much to the chagrin of
then Viceroy Lord Curzon — regarded among the most vituperative of racists to occupy that post.
In 1899, Gokhale joined the Indian National Congress, emerging as one of the main leaders of its ‘moderate’
wing, and gave up teaching three years later to work as a lawmaker for the remainder of his life.
Source: Indias struggle for Independence by Bipan Chandra, Chapter: Propaganda in the legislatures.
https://indianexpress.com/article/who-is/who-was-gopal-krishna-gokhale-liberal-nationalist-mahatma-
gandhi-6403225/
Sub) History, Modern History, Gopal Krishna Gokhale

Q.16)
Ans) a
Exp) option a is correct.
Statement 1 is correct. The supreme court has held that the freedom of speech and expression and the
freedom to practice any profession or carry on any trade, business or occupation over the medium of
internet enjoys constitutional protection under article 19 (1).
Statement 2 is incorrect. Kerala had become the first state in 2017 to declare access to Internet "a basic
human right".
Source: https://www.indiatoday.in/news-analysis/story/internet-access-fundamental-right-supreme-
court-makes-official-article-19-explained-1635662-2020-01-10
Sub) Polity, Indian Constitution, Right to internet access is a FR

Q.17)
Ans) a
Exp) Option a is correct.
Statement 1 is correct. Voters under preventive detention can vote only by postal ballot. Members of the
armed forces like the Army, Navy and Air Force, members of the armed police force of a state. (serving
outside the state), government employees posted outside India and their spouses are also entitled to vote
only by post. They can’t vote in person.
Statement 2 is correct. Postal ballots are counted first on the counting day of an election. The postal ballot
should reach the returning officer by 8 am on the day of the counting. Counting of votes begin with
counting of postal ballots at 8 am.
Statement 3 is incorrect. Non-Resident Indians (NRIs) were not allowed to vote through postal ballot in
2019 parliamentary general election. It was just in November, 2020, that the Election Commission had
written to the Law Ministry with the proposal of extending postal ballots to overseas electors including
Non-Resident Indians (NRIs). It is not yet implemented.
Source: https://www.thehindu.com/news/national/tamil-nadu/here-is-how-you-can-cast-a-postal-
vote/article33984268.ece
Sub) Polity, Indian Constitution, Postal Vote

ForumIAS Offline Guidance Centre


2nd Floor, IAPL House, 19, Pusa Road, Karol Bagh, New Delhi – 110005 | helpdesk@forumias.academy|9821711605
Page 8 of 42

PTS 2021|Simulator Test 3– Solutions|ForumIAS


Q.18)
Ans) b
Exp) Option b is correct.
Statement a is incorrect. Agreement to create the African Continental Free Trade Area (AfCFTA) was
signed in Rwanda, on March 21, 2018. Trading under the AfCFTA has started from 1st January, 2021. Till
date 35 of the African Union’s (AU) 55 member States have ratified the agreement and 54 AU member
States have signed the AfCFTA, with the exception of Eritrea.
Statement b is correct. The free-trade area is the largest in the world in terms of the number of participating
countries since the formation of the World Trade Organization. It is the largest regional free trade
agreement in terms of number of participating countries.
Statement c is incorrect. There is another free trade area. The African Free Trade Zone (AFTZ) was
established in 2008 by the heads of Southern African Development Community (SADC), the Common
Market for Eastern and Southern Africa (COMESA) and the East African Community (EAC).
Statement d is incorrect. AfCFTA has not subsumed the existing regional economic communities. Intra-
African trade will continue on multiple tracks like the Southern African Customs Union (SACU) and the
Southern African Development Community (SADC). As the AfCFTA advances and becomes more
consolidated, there would be more policy convergence, and a simplification of rules across the different
trading regimes.
Source: https://www.weforum.org/agenda/2021/02/afcfta-africa-free-trade-global-game-changer/
Sub) International Relations, Agreements / Conventions / Declarations, African Continental Free trade
Area

Q.19)
Ans) d
Exp) Option d is correct.
Option a is incorrect. Rushikonda beach is widely known for its golden sands and tidy waves of Bay of
Bengal. It is located in Visakhapatnam, Andhra Pradesh. It has been given Blue Flag certification recently.
Option b is incorrect. Radhanagar Beach is one of the most famous attractions and is situated at Havelock
Island in Andaman and Nicobar Islands. It has been given Blue Flag certification recently.
Option c is incorrect. Kappad beach is situated in Kozhikode, Kerala. It has historical importance also as
Vasco da Gama first landed on the shores of Kappad. It has been given Blue Flag certification recently.
Option d is correct. Recently Padubidri beach has been accorded ‘Blue Flag’ tag by Foundation for
Environment Education (FEE) for having grey water treatment plants, solid waste management plants,
disabled-friendly equipment to enable them to enter seawater, clean drinking water, bathing facility,
disabled-friendly and general toilets, solar power plant, solar lighting. It is situated in Karnataka and is
famous for Dakkebali ritual held once in two years.
Knowledge Base:

ForumIAS Offline Guidance Centre


2nd Floor, IAPL House, 19, Pusa Road, Karol Bagh, New Delhi – 110005 | helpdesk@forumias.academy|9821711605
Page 9 of 42

PTS 2021|Simulator Test 3– Solutions|ForumIAS

Source: https://www.thehindu.com/news/cities/Mangalore/padubidri-end-point-beach-thrown-open-to-
visitors/article33358954.ece
https://www.karnataka.com/udupi/padubidri-beach/
Sub) Geography, Indian Physical Geography, Famous beach in news (Avoid beach asked in 2017 prelims
paper)

Q.20)
Ans) a
Exp) Option a is correct.
Statement 1 is correct. The minister of labour had claimed that existing labour laws only governed payment
of wages for about 40% of the labour force and the Code on Wages would extend coverage to the entire
labour force irrespective of sector or wage ceiling.
Statement 2 is incorrect. Wages include salary, allowance, or any other component expressed in monetary
terms. This does not include bonus payable to employees or any travelling allowance, among others.
Source: https://www.theleaflet.in/labour-codes-govt-defers-implementation-with-states-not-having-
finalised-rules/
https://www.prsindia.org/theprsblog/decoding-code-wages
Sub) Polity, Bills & Acts, Labour code

Q.21)
Ans) a
Exp) Option a is correct
Relevance: UNESCO’s Executive Board has approved the designation of eight new UNESCO Global
Geoparks, which brings the number of sites participating in the Global Geoparks Network to 169 in 44
countries.
Statement 1 is correct: UNESCO Global Geoparks are single, unified geographical areas where international
geologically significant sites and ecosystems are managed with a holistic approach to security, education,
and sustainable development.
Statement 2 is correct: A UNESCO Global Geopark is given this designation for a period of four years after
which the functioning and quality of each UNESCO Global Geopark is thoroughly re-examined during a
revalidation process.
Statement 3 is incorrect: Though there are 161 UNESCO Global Geoparks in 44 countries, India has yet to
have any designated sites as global geoparks.

ForumIAS Offline Guidance Centre


2nd Floor, IAPL House, 19, Pusa Road, Karol Bagh, New Delhi – 110005 | helpdesk@forumias.academy|9821711605
Page 10 of 42

PTS 2021|Simulator Test 3– Solutions|ForumIAS


Source: http://www.unesco.org/new/en/natural-sciences/environment/earth-sciences/unesco-global-
geoparks/list-of-unesco-global-geoparks/
Sub) Environment, Biodiversity, UNESCO Global Geoparks

Q.22)
Ans) b
Exp) Option b is correct.
Statement 1 is correct. The Genetic Engineering Appraisal Committee (GEAC) functions in the Ministry of
Environment, Forest and Climate Change (MoEF&CC).
Statement 2 is correct. The Genetic Engineering Appraisal Committee (GEAC) is a statutory body
constituted under the ‘Rules for the Manufacture, Use /Import /Export and Storage of Hazardous
Microorganisms/ Genetically Engineering Organisms or Cells, 1989 notified under the Environment
(Protection) Act, 1986.
Statement 3 is incorrect. GEAC is chaired by the Special Secretary/Additional Secretary of MoEF&CC and
co-chaired by a representative from the Department of Biotechnology (DBT).
Knowledge Base: As per Rules, 1989, it is responsible for appraisal of activities involving large scale use of
hazardous microorganisms and recombinants in research and industrial production from the
environmental angle. The committee is also responsible for appraisal of proposals relating to release of
genetically engineered (GE) organisms and products into the environment including experimental field
trials.
GEAC Functions
The GEAC is the apex biotech regulatory body in India. It is a statutory body. The functions of the GEAC
are described below.
1) It is responsible for the appraisal of activities that involve the large-scale use of hazardous microbes and
recombinants in research and industrial production from the point of view of the environment.
2) The GEAC also assesses proposals regarding the release of genetically engineered products and
organisms into the environment, and this includes experimental field trials as well.
3) The body also looks into proposals regarding the use of living modified organism that comes in the risk
category III and above in the import/manufacture of recombinant pharma products, or where the end-
product of the recombinant pharma product is a modified living organism.
4) The Committee has the power to take punitive action against people/body under the Environment
(Protection) Act.
5) The approval of the GEAC is mandatory before genetically modified organisms and products derived
from them can be used commercially.
Source: https://geacindia.gov.in/about-geac-india.aspx
http://www.geacindia.gov.in/capacity-building-resources.aspx
Sub) Science and Technology, Bio Technology, GEAC

Q.23)
Ans) b
Exp) Option b is correct.
Statement 1 is correct: The provisions that require the prior permission of the government for lay-off,
retrenchment and closure are made applicable to only establishments that had employed 300 or more
workers on an average per working day in the preceding 12 months. The Code also allows the government to
raise this threshold by notification.

ForumIAS Offline Guidance Centre


2nd Floor, IAPL House, 19, Pusa Road, Karol Bagh, New Delhi – 110005 | helpdesk@forumias.academy|9821711605
Page 11 of 42

PTS 2021|Simulator Test 3– Solutions|ForumIAS


Statement 2 is correct: Where there is more than one trade union in an establishment, the sole negotiating
union status will be given to the one that has 51% of the employees as its members. Where no union qualifies
under this criterion, the employer must constitute a ‘negotiating’.
council’ consisting of representatives drawn from the various unions, with only those with at least 20% of
employees as its members.
Statement 3 is incorrect: Fixed term employment refers to workers employed for a fixed duration based on a
contract signed between the worker and the employer. The Code does not restrict the type of work in which
fixed term workers may be hired. Therefore, they may be hired for roles offered to permanent workmen.
Statement 4 is correct: The 2020 Code provides that women will be entitled to be employed in all
establishments for all types of work under the Bill. It also provides that in case they are required to work in
hazardous or dangerous operations, the government may require the employer to provide adequate
safeguards prior to their employment. Source: https://prsindia.org/billtrack/the-industrial-relations-code-
2020
Sub) Economics, Schemes, Labour codes – reforms

Q.24)
Ans) c
Exp) Option c is correct.
Statement 1 is correct. After C.R Das death the Congress leadership in Bengal got divided into two wings,
one led by Subhas Chandra Bose and the other by J.M. Sengupta, the Yugantar group joined forces with the
first and Anushilan with the second.
Statement 2 is correct. In the beginning, the activities of samiti were limited to giving physical and moral
training to the members and remained insignificant till 1907-08. In 1906, an inner circle within Anushilan
(Barindra Kumar Ghosh, Bhupendranath Dutta) started the weekly Yugantar, which openly preached
armed rebellion in order to create the necessary revolutionary mentality among the people. It openly
preached the cult of violence.
Source: Modern History, Spectrum, Chapter-13, Pg. 283,284
https://indianexpress.com/article/cities/kolkata/kolkata-five-spots-linked-to-the-freedom-struggle-you-
must-know-about-5906868/
India’s Struggle for independence by Bipan Chandra, Ch: Bhagat Singh, Surya Sen and the revolutionary
terrorists.
Sub) History, Modern History, Anushilan samite

Q.25)
Ans) a
Exp) Option a is correct
Statement 1 is correct: The Global Warming Potential (GWP) was developed to allow comparisons of the
global warming impacts of different gases. Specifically, it is a measure of how much energy the emissions of
1 ton of a gas will absorb over a given period of time, relative to the emissions of 1 ton of carbon dioxide
(CO2). Nitrous Oxide (N2O) has a GWP 265–300 times that of CO2 for a 100-year timescale. N2O emitted
today remains in the atmosphere for more than 100 years, on average.
Statement 2 is correct: Nitrous oxide emission from agricultural soil is due to the natural biochemical
process in the nitrogen cycle.
Statement 3 is incorrect: Nitrous oxide has the third-highest concentration — after CO2 and methane — in
the atmosphere among greenhouse gases responsible for global warming. N2O can live in the atmosphere
for up to 125 years.

ForumIAS Offline Guidance Centre


2nd Floor, IAPL House, 19, Pusa Road, Karol Bagh, New Delhi – 110005 | helpdesk@forumias.academy|9821711605
Page 12 of 42

PTS 2021|Simulator Test 3– Solutions|ForumIAS


Source: https://scroll.in/article/924221/nitrous-oxide-emission-from-agriculture-has-
risen-sharply-over-30-years-finds-study
Sub) Environment, Climate Change, N2O emissions

Q.26)
Ans) c
Exp) Option c is correct.
Option c is incorrect. In 1902, Raleigh Commission was set up to go into conditions and prospects of
universities in India and to suggest measures for improvement in their constitution and working. Based on
its recommendations, the Indian Universities Act was passed in 1904. The act placed Calcutta University
under the governmental control. Infact all the universities were brought under governmental control.
As per the Act,
1) Universities were to give more attention to study and research.
2) Government was to have powers to veto universities’ senate regulations and could amend these
regulations or pass regulations on its own.
3) Affiliation of private colleges was not encouraged. In fact, conditions were to be made stricter for
affiliation of private colleges. Knowledge Base:
Other features of Indian Universities Act, 1904:
1) The number of fellows of a university and their period in office were reduced and most fellows were to
be nominated by the Government.
2) Five lakh rupees were to be sanctioned per annum for five years for improvement of higher education
and universities.
3) The act was passed during the tenure of Lord Curzon.
Source: Modern History, Spectrum, 6th Edition, Chapter-30, Pg. 567-568
From Plassey to Partition, Sekhar Bandyopadhyay, Chapter-5, Pg. 249
Sub) History, Modern History, Indian Universities Act 1904 - Higher education reforms

Q.27)
Ans) b
Exp) Option b is correct.
Option a is incorrect: Slowdown is the situation in which GDP growth slows but does not decline. For
example, if GDP goes from 5% growth to 3% growth.
Option b is correct: Contraction in Gross domestic Product (GDP) for a second consecutive quarter means
the economy is in a ‘technical recession’.
Option c is incorrect: Economic depression is a sustained, long-term downturn in economic activity. IT is
more severe than recession.
Option d is incorrect: Economic stagnation is a prolonged period of slow economic growth.
Source: https://indianexpress.com/article/explained/what-is-a-technical-recession-indian-economy-
coronavirus-7049758/
Sub) Economics, Indian Economy, Technical recession

Q.28)
Ans) a
Exp) Option a is correct.
Statement 1 is correct. The Asia-Pacific Economic Cooperation (APEC) is a regional economic forum
established in 1989 to leverage the growing interdependence of the Asia-Pacific. Its membership includes 21

ForumIAS Offline Guidance Centre


2nd Floor, IAPL House, 19, Pusa Road, Karol Bagh, New Delhi – 110005 | helpdesk@forumias.academy|9821711605
Page 13 of 42

PTS 2021|Simulator Test 3– Solutions|ForumIAS


economies including Taiwan, China, Hong Kong, The United States of America, Canada, Mexico, Australia
and New Zealand.
Statement 2 is incorrect. The headquarter is situated in Singapore.
Source: https://www.brookings.edu/research/time-to-reinvent-apec/
Sub) International Relations, Multilateral Organisations, APEC

Q.29)
Ans) c
Exp) Option c is the answer.
Option a is correct: State-owned banks plan to transfer nearly ₹2 trillion of bad loans to the ARC without
going through any price discovery mechanism, according to the proposal. In return, the ARC will provide
15% upfront cash to the banks and issue security receipts for the remaining 85% to be guaranteed by the
government.
Option b is correct: The ARC, which will have an Asset Management Company (AMC) to manage and sell
bad assets, will look to resolve stressed assets of Rs 2-2.5 lakh crore.
Option c is incorrect: There will be no equity contribution from the government. The proposed ARC will
need capital infusion of at least ₹10,000 crore from banks.
Option d is correct: The ARC by resolving bad loans will help in cleaning the balance sheets of banks.
Source: https://www.livemint.com/news/india/proposed-bad-bank-not-to-jeopardise-existing-players-
rbi-governor-11614259042940.html
https://indianexpress.com/article/business/banking-and-finance/arc-to-have-no-govt-equity-psbs-pvt-
banks-to-set-it-up-7171968/
Sub) Economics, Schemes, Asset Reconstruction Company - Bad Banks

Q.30)
Ans) c
Exp) Option c is correct.
Statement 1 is incorrect. River Tapi originates near Multai reserve forest in Betul district of Madhya
Pradesh.
Statement 2 is correct. The important tributaries of Tapi river are the Suki, the Gomai, the Arunavati and
the Aner which joins it from right, and those joining from left are the Vaghur, the Amravati, the Buray, the
Panjhra, the Bori, the Girna, the Purna, the Mona and the Sipna.
Statement 3 is correct. River Tapi flows through rift valley. It also flows from west to east. (River Narmada
and Tapi both flow through the rift valley)
Statement 4 is incorrect. For the first 282 Km, the river flows in Madhya Pradesh, out of which 54 Km
forms the common boundary with Maharashtra State. It flows for 228 Km in Maharashtra draining
Khandesh and east Vidarbha regions in the northwest corner of the Deccan Plateau before entering Gujarat.
Source: https://indiawris.gov.in/wiki/doku.php?id=tapi
https://ncert.nic.in/ncerts/l/kegy103.pdf
Sub) Geography, Indian Physical Geography, Tapti river profile

Q.31)
Ans) a
Exp) Option a is correct.
Statement 1 is correct. ASF is a severe viral disease that affects wild and domestic pigs typically resulting in
an acute hemorrhagic fever. The disease has a case fatality rate (CFR) of almost 100 per cent. Its routes of

ForumIAS Offline Guidance Centre


2nd Floor, IAPL House, 19, Pusa Road, Karol Bagh, New Delhi – 110005 | helpdesk@forumias.academy|9821711605
Page 14 of 42

PTS 2021|Simulator Test 3– Solutions|ForumIAS


transmission include direct contact with an infected or wild pig (alive or dead), indirect contact through
ingestion of contaminated material such as food waste, feed or garbage, or through biological vectors such
as ticks.
Statement 2 is incorrect. African Swine Fever (ASF) does not affect humans but can be catastrophic for
pigs. It is a notifiable, non-zoonotic disease for which no effective treatment or vaccine currently exists.
Statement 3 is incorrect. The current outbreak of ASF in India (in Assam) is the first time that the disease
has been reported in the country.
Knowledge Base: Zoonoses or Zoonotic Disease:
1) It is a disease that passes into the human population from an animal source directly or through an
intermediary species.
2) Zoonotic infections can be bacterial, viral, or parasitic in nature, with animals playing a vital role in
maintaining such infections.
3) Examples of zoonotic diseases include HIV-AIDS, Ebola, Malaria, and the current Covid-19 disease.
Source: https://indianexpress.com/article/explained/african-swine-fever-assam-china-6523760/
https://www.boehringer-ingelheim.us/news/news/african-swine-fever-what-you-need-know-5-
questions-and-answers
https://www.oie.int/doc/ged/D11831.PDF
Sub) Science and Technology, Diseases and Vaccine, African swine flu- first outbreak in India

Q.32)
Ans) b
Exp) Option b is correct.
Statement 1 is incorrect: The VRR channel is aimed at attracting long-term and stable FPI investments into
debt markets. VRR scheme allows FPIs to participate in repos and also invest in exchange traded funds that
invest in debt instruments.
Statement 2 is correct: The VRR route was introduced in March 2019 wherein FPIs have to invest the
amount allocated, called the committed portfolio size (CPS), in relevant debt instruments and remain
invested at all times during the voluntary retention period. The minimum retention period was kept at three
years.
Source: https://indianexpress.com/article/business/economy/fpis-pause-selloff-mode-outflow-drops-88-
in-april-6392367/
https://www.financialexpress.com/market/fpis-get-more-time-to-invest-under-voluntary-retention-
route/1968144/
Sub) Economics, Schemes, FPIs – definition

Q.33)
Ans) a
Exp) Option a is correct.
Statement 1 is correct. The National Financial Reporting Authority (NFRA) is a body constituted under the
provisions of Section 132 of the Companies Act, 2013. The constitution of this authority is effective from 1st
October 2018. The aim of the Central Government is:
1) Setting up of a separate and independent regulatory body to assist in the framing and enforcement of
legislation relating to accounting & auditing and
2) Improving investor and public confidence in the financial reporting of an entity.
3) Supposedly, the need for this authority arose as a response to various corporate scams in recent times.

ForumIAS Offline Guidance Centre


2nd Floor, IAPL House, 19, Pusa Road, Karol Bagh, New Delhi – 110005 | helpdesk@forumias.academy|9821711605
Page 15 of 42

PTS 2021|Simulator Test 3– Solutions|ForumIAS


Statement 2 is incorrect. It can probe listed companies and those unlisted public companies having paid-up
capital of no less than Rs 500 crore or annual turnover of no less than Rs 1,000 crore.
It can investigate professional misconduct committed by members of the Institute of Chartered
Accountants of India (ICAI) for prescribed class of body corporate or persons.
Statement 3 is incorrect. In cases where professional or other misconduct is proved, NFRA has the power to
impose penalty of different amounts and can also debar the member or firm from practicing as member of
Institute of Chartered Accountant of India.
Source: https://cleartax.in/s/nfra
https://nfra.gov.in/about-us
https://www.mca.gov.in/SearchableActs/Section132.htm
Sub) Economics, Schemes, NFRA

Q.34)
Ans) d
Exp) Option d is correct.
Statement 1 is incorrect. Refer below graph.

Statement 2 is incorrect. The Central Issue price of rice and wheat for National food security act
beneficiaries has not been revised since the introduction of the act in 2013.
Source: Economic Survey Volume 2 2021 Chapter 7 Pg. 251, 252
Sub) Economics, Indian Economy, Subsidies trend - Budget/ES

Q.35)
Ans) d
Exp) Option d is correct.
Statement 1 is incorrect. India has 1 permanent research station in Arctic named as Himadri.
Statement 2 is incorrect. India had in the past established Dakshin Gangotri (1983) and Maitri (1988-89)
stations in Antarctica. The former has since been decommissioned after it got buried under ice and has been
marked as an historic site. India established 3rd research station Bharati in 2015.
Statement 3 is incorrect. India has been elected as an observer of Arctic council for the first time in 2013.
India was re-elected as an observer status in 2019 for the second time.
Source: https://indianexpress.com/article/explained/india-re-election-observer-arctic-council-
importance-5727126/ https://www.thehindu.com/sci-tech/science/india-to-commission-third-research-
station-in-antarctica/article2606767.ece
Sub) Environment, Convention / Organisation/Laws/Protocols, tiger reserves- ‘core – buffer’

ForumIAS Offline Guidance Centre


2nd Floor, IAPL House, 19, Pusa Road, Karol Bagh, New Delhi – 110005 | helpdesk@forumias.academy|9821711605
Page 16 of 42

PTS 2021|Simulator Test 3– Solutions|ForumIAS


Q.36)
Ans) b
Exp) Option b is correct.
Statement 1 is incorrect: The presiding officer (The Speaker for Lok Sabha, The Chairman for Rajya Sabha)
of the house nominates member of the Committee on Ethics.
Statement 2 is correct: The committee examine cases concerning the alleged breach of the Code of Conduct
by members as also cases concerning allegations of any other ethical misconduct of member.
Statement 3 is incorrect: The committee tenders advice to members from time to time on questions
involving ethical standards either Suo motu or on receiving specific requests.
Source: https://rajyasabha.nic.in/rsnew/practice_procedure/book16.asp
Sub) Polity, Indian Constitution, Parliamentary committee - ethics committee

Q.37)
Ans) d
Exp) Option d is correct.
Pair 1 is correctly matched. During Warren Hastings Governor Generalship through the Regulating Act of
1773, a Supreme Court was established at Calcutta which was competent to try all British subjects within
Calcutta and the subordinate factories, including Indians and Europeans.
Pair 2 is correctly matched. During the governor generalship of Lord Cornwallis (1786-93), the District
Fauzdari Courts were abolished and, instead, circuit courts were established which were to act as courts of
appeal for both civil and criminal cases.
Pair 3 is correctly matched. A Law Commission was set up in 1833 under Macaulay for codification of
Indian laws under the reign of William Bentinck (1828-1833). As a result, a Civil Procedure Code (1859), an
Indian Penal Code (1860) and a Criminal Procedure Code (1861) were prepared. Also in the Supreme Court,
English language replaced Persian.
Knowledge Base:
Other judicial reforms associated with Warren Hastings (1772-1785) are: District Diwani Adalats were
established in districts to try civil disputes. These adalats were placed under the collector. District Fauzdari
Adalats were set up to try criminal disputes and were placed under an Indian officer assisted by qazis and
muftis. These adalats also were under the general supervision of the collector.
Other judicial reforms associated with Cornwallis are: The Sadar Nizamat Adalat was shifted to Calcutta
and was put under the governor-general. The collector was made now responsible only for the revenue
administration with no magisterial functions. A gradation of civil courts was established (for both Hindu
and Muslim laws).
Source: Modern History, Spectrum, Chapter-26, Pg. 521-523
Sub) History, Modern History, Judicial reforms

Q.38)
Ans) d
Exp) Option d is correct.
Statement is incorrect. The Constitution has been framed by the ‘people of India’ assembled in the
Constituent Assembly, and the Union of India is not a product of any compact or agreement between
autonomous States.
Option b is incorrect. Our Constitution has followed the Canadian system with respect to residuary powers.
In India, under the Seventh Schedule residuary powers are vested in the Union government.

ForumIAS Offline Guidance Centre


2nd Floor, IAPL House, 19, Pusa Road, Karol Bagh, New Delhi – 110005 | helpdesk@forumias.academy|9821711605
Page 17 of 42

PTS 2021|Simulator Test 3– Solutions|ForumIAS


Option c is incorrect. Under our Constitution, it is not possible for the States of the Union of India, to
exercise any right of secession. But the Union Parliament can reorganise the states or alter their boundaries,
by a simple majority in the ordinary process of legislation. For this, the Constitution does not require the
consent of the states. Hence, Indian federalism is called as the indestructible Union composed of
destructible States.
Option d is correct. In India, there is supremacy of the Constitution. In a federal State, the legal supremacy
of the Constitution is essential to the existence of the federal system. It is essential to maintain the division
of powers not only between the coordinate branches of the government, but also between the Federal
Government and the States themselves.
Source: Indian Polity by M. Laxmikanth. Chapter 13. The Federal system
Sub) Polity, Indian Constitution, Federalism – concept

Q.39)
Ans) c
Exp) Option c is correct.
Statement 1 is incorrect: In the first three phases the Centre procured the entire quantity of vaccines from
the manufacturers, Serum Institute of India (Covishield) and Bharat Biotech (Covaxin), and distributed it
to states. The states distributed the stock to government vaccination centres and to private hospitals.
For the fourth phase, under the new vaccine policy the supply of vaccine will be divided into two baskets: 50
per cent for the Centre, and 50 per cent for the open market.
Through the second channel, state governments, private hospitals, and industries that have facilities to
administer the vaccine, will be able to procure doses directly from manufacturers.
Statement 2 is incorrect. The government would not make vaccine available to private hospitals. They
would procure it from the ‘Open Market’ using the second (non-government) channel. Private hospitals
will have their own rates.
Statement 3 is correct. No other countries in the world have allowed open-market sale of vaccines so far.
The main reason is that the vaccines that are being used around the world have received only Emergency
Use Authorisation (EUA), none of them have presented enough evidence yet on their safety and
effectiveness to receive full regulatory authorisation.
Source: https://indianexpress.com/article/explained/india-vaccine-policy-covid-19-serum-covaxin-
7283776/
Sub) Polity, Governance & Social Justice, Vaccine policy of india

Q.40)
Ans) b
Exp) Option b is correct.
Statement 1 is incorrect. Unlike Patent, copyright does not specifically require registration. Copyright does
not mandatorily require registration but registration proves originality.
Statement 2 is correct. IP rights (Patents, Copyrights etc) are territorial in nature. It means that an Indian
registration is valid only in India. For protection of Intellectual Property in any other country, one must seek
protection separately under the relevant law.
Knowledge Base) Copyright is a tool to protect intellectual property which can be in the form of literature,
sound recordings, painting or sculptures, books or poetry or screenplays etc. For an individual owner, the
term of copyright is the lifetime of the author plus 60 years whereas for non-individual owner, a copyright
may last upto 60 years from the date of the publication.

ForumIAS Offline Guidance Centre


2nd Floor, IAPL House, 19, Pusa Road, Karol Bagh, New Delhi – 110005 | helpdesk@forumias.academy|9821711605
Page 18 of 42

PTS 2021|Simulator Test 3– Solutions|ForumIAS


Patent is an exclusive right or a legal document which is granted when the invention in question is found to
be new, novel, contains inventive steps and has industrial applicability. The patent is applicable for 20 years
from the date of application for which renewal fees is required to be paid every year.
Source) https://www.meity.gov.in/content/faqs-
ipr#:~:text=Are%20IP%20rights%20territorial%20in,separately%20under%20the%20relevant%20law.
https://www.mondaq.com/india/trademark/603960/difference-between-trademarks-copyrights-and-
patents
Sub) Economics, Indian Economy, Patents/copy rights

Q.41)
Ans) d
Exp) Option d is correct.
Statement 1 is incorrect. The Agreement applies to the moon as well as other celestial bodies. This
Agreement also include orbits or other trajectories around the moon and the celestial bodies. It does not
apply to extra-terrestrial materials which reach the surface of the earth by natural means.
Features:
1) The agreement prohibits the use of the Moon in order to commit any hostile act or to engage in any such
threat in relation to the Earth.
2) The Moon and its natural resources are the common heritage of mankind. The Moon is not subject to
national appropriation by any claim of sovereignty, by means of use or occupation, or by any other
means.
3) Neither the surface nor the subsurface of the Moon, nor any part thereof or its natural resources, can
become the property of any Country, Organization or person.
4) The placement of personnel, space vehicles, equipment, facilities, stations and installations on or below
the surface of the Moon shall not create a right of ownership.
Statement 2 is incorrect. It has not been ratified by any state that engages in self-launched human
spaceflight (e.g. the United States, Russia (former Soviet Union), People's Republic of China). India has
signed but has not ratified the treaty.
Source: https://www.thehindubusinessline.com/news/science/why-india-should-exit-the-moon-
agreement/article31634373.ece
Sub) International Relations, Agreements / Conventions / Declarations, Moon treaty

Q.42)
Ans) b
Exp) Option b is correct.
Statement 1 is incorrect. Article 102 provides that a person shall be disqualified for being chosen as, and for
being, a member of either House of Parliament if he holds any office of profit under the Government of
India. However, the constitution does not define the term ‘office of profit’.
In the Guru Gobinda Basu vs Sankari Prasad Ghosal (1964) case, the Supreme Court observed that the test
for determining whether a person holds an office of profit is the test of appointment.
Several factors are considered in this determination including factors such as:
(1) whether the government is the appointing authority
(2) whether the government has the power to terminate the appointment
(3) whether the government determines the remuneration
(4) what is the source of remuneration,
(5) the power that comes with the position.

ForumIAS Offline Guidance Centre


2nd Floor, IAPL House, 19, Pusa Road, Karol Bagh, New Delhi – 110005 | helpdesk@forumias.academy|9821711605
Page 19 of 42

PTS 2021|Simulator Test 3– Solutions|ForumIAS


Similar view was upheld in Pradyut Bordoloi vs Swapan Roy (2001) case.
Statement 2 is correct. A person who acquires a contract or licence from a government to perform
functions, which the government would have itself discharged, will not be held guilty of holding an office
of profit. So, acquiring a gas agency from the government or holding a permit to ply do not amount to
holding office of profit.
Statement 3 is correct. Parliament can declare by law that the holding of certain offices will not incur this
disqualification. The Parliament (Prevention of Disqualification) Act, 1959 lists certain offices of profit
under the central and state governments, which do not disqualify the holders from being a member of
Parliament. The act has been amended several times to expand the exempted list of certain offices from the
purview of office of profit.
Source: https://theprint.in/india/governance/defining-office-of-profit-not-a-good-idea-law-ministry-
tells-parliamentary-panel/547270/
Sub) Polity, Indian Constitution, Office of profit

Q.43)
Ans) b
Exp) Option b is correct.
Top 4 countries in steel production in 2018 and 2019 were China, India, Japan, USA respectively.
India became the second largest steel producer of crude steel after China in 2018 and 2019, by replacing
Japan.
In 2020, while India retained its second spot, its production fell by 10.6% in 2020 to 99.6 MT from 111.4 MT
a year earlier. India’s share in the global output shrunk to 5.3% in 2020 from 5.9% a year ago. Japan also
retained its third spot, but its production fell by 16.2% to 83.2 MT.
Source: https://pib.gov.in/PressReleasePage.aspx?PRID=1602023
https://www.financialexpress.com/industry/indias-steel-production-declines-by-more-than-10/2180201/
Sub) Economics, Indian Economy, Steel production and India - export issues

Q.44)
Ans) b
Exp) Option b is correct.
Concave lens is thicker at the edges than at the middle. Such lenses diverge light rays. Such lenses are also
called diverging lenses. A double concave lens is simply called a concave lens. A concave lens will always give
a virtual, erect and diminished image, irrespective of the position of the object.
Myopia which is also known as near-sightedness can be corrected by using a concave lens of suitable power.
A concave lens of suitable power will bring the image back on to the retina and thus the defect is corrected.
As a concave lens gives diminished image of the object, so it gives view of the wider area. This makes
concave lens suitable for the peepholes usually installed in the home doors.
Source: NCERT, Class X, Science, Chapter 10, Page 176, 179
NCERT, Class X, Science, Chapter 11, Page 189
Sub) Science and Technology, Physics, Different kinds of lenses - reflection/refraction

Q.45)
Ans) d
Exp) Option d is correct.

ForumIAS Offline Guidance Centre


2nd Floor, IAPL House, 19, Pusa Road, Karol Bagh, New Delhi – 110005 | helpdesk@forumias.academy|9821711605
Page 20 of 42

PTS 2021|Simulator Test 3– Solutions|ForumIAS


3D Bioprinting is a form of additive manufacturing that uses cells and other biocompatible materials as
“inks”, also known as bioinks, to print living structures layer-by-layer which mimic the behavior of natural
living systems.
Option 1 is correct – Scientists have developed a new 3D bioprinter that can create totally functional human
skin, which can be used for transplants as well as testing of cosmetic and pharmaceutical products. It
replicates the natural structure of the skin, with a first external layer, the epidermis with its stratum
corneum, which acts as protection against the external environment, together with another thicker, deeper
layer, the dermis. This last layer consists of fibroblasts that produce collagen, the protein that gives elasticity
and mechanical strength to the skin.
Option 2 is correct – Researchers from Indian Institute of Technology (IIT) Delhi and IIT Kanpur have
used a different approach to mimic the development biology pathway by which adult load-bearing, long
bones are formed. The bone construct was fabricated by combining tissue engineering and 3D bioprinting.
The cartilage was first 3D bioprinted using bioink and cartilage characteristics were thoroughly
characterised. The researchers then added a thyroid hormone (Triiodothyronine or T3) to the cartilage to
facilitate the differentiation of cartilage into bone-like cells.
Option 3 is correct – The Sree Chitra Tirunal Institute for Medical Sciences and Technology (SCTIMST)
has already developed bioinks to print functional liver tissues from hepatocytes. These artificial tissues can
be used in the pharma industry for drug screening to test liver toxicity.
Option 4 is correct - 3D bioprinting could help eliminate the need for testing new drugs on animals thus
replacing lengthy and expensive clinical trials. 3D printed tissue prove to be an effective means of testing
new pharmaceuticals. Thus, the drug could be thoroughly assessed and brought to market more quickly.
Knowledge Base: Developments in 3D bioprinting have been mostly motivated by the limited availability of
organs globally, which are needed for the rehabilitation of lost or failed organs and tissues. The most
challenging and demanding applications for engineered tissues include the skin, cartilage, hard tissues such
as bones, cardiac tissue, and vascular grafts.
Source: https://indianexpress.com/article/technology/tech-news-technology/new-3d-bioprinter-can-
create-functional-human-skin-4490625/
https://www.thehindu.com/sci-tech/iit-researchers-3d-bioprint-load-bearing-bones/article25752356.ece
https://www.thehindu.com/news/cities/Thiruvananthapuram/sree-chitra-acquires3d-bioprinting-
facility/article25340532.ece
https://www.ncbi.nlm.nih.gov/pmc/articles/PMC6533576/
https://medicalfuturist.com/3d-bioprinting-overview/
Sub) Science and Technology, Bio Technology, 3 D bioprinting - functional liver tissue in Chennai

Q.46)
Ans) d
Exp) Option d is correct.
‘The Haryana Panchayati Raj (second amendment) Act 2020’ contains provision for recall of an elected
sarpanch reflecting his accountability to the gram sabha. The Amendment also provides 50% reservation to
women in panchayati raj institutions and 8% reservation to persons from backward class (A) for the post of
sarpanch.
Source: https://www.thehindu.com/news/national/other-states/haryana-clears-bill-on-right-to-recall-
panchayat-member/article33044214.ece
Sub) Polity, Indian Constitution, Right to recall

ForumIAS Offline Guidance Centre


2nd Floor, IAPL House, 19, Pusa Road, Karol Bagh, New Delhi – 110005 | helpdesk@forumias.academy|9821711605
Page 21 of 42

PTS 2021|Simulator Test 3– Solutions|ForumIAS


Q.47)
Ans) c
Exp) Option c is correct.
Option a is incorrect. A preamble is an introductory statement in a document that explains the document’s
philosophy and objectives. The Preamble to India’s Constitution is based on the ‘Objectives Resolution’,
adopted by the Constituent Assembly on January 22, 1947.
Option b is incorrect. In the Keshavananda Bharati vs state of Kerala, 1973 case the Supreme court held that
the Preamble is neither a source of power to parliament nor a prohibition upon the powers of the
parliament.
Option c is correct. The words, “We, the people of India…” in the preamble indicate the ultimate
sovereignty of the people of India. Sovereignty means the independent authority of the State, not being
subject to the control of any other State or external power.
Option d is incorrect. Also in the Union Government Vs LIC of India, 1995, the Supreme Court has held
that Preamble is the integral part of the Constitution but is not directly enforceable in a court of justice in
India. However, the Preamble states the objects of the Constitution, and acts as an aid during the
interpretation of Articles when language is found ambiguous.
Source: Indian Polity by M. Laxmikanth. Chapter 4. Preamble of the Constitution
Sub) Polity, Indian Constitution, Preamble - Constitutional morality

Q.48)
Ans) d
Exp) Option d is correct.
Statement a is incorrect. The Constitution empowers the Parliament to make laws on any matter
enumerated in the State List in following circumstances –
1) When Rajya Sabha Passes a Resolution
2) During a National Emergency
3) When States Make a Request
4) To Implement International Agreements
5) During President’s Rule
Statement b is incorrect. Centre will levy and administer CGST and IGST while respective states will levy
State-GST.
Statement c is incorrect. According to the Constitution there is predominance of the Union List over the
State List and the Concurrent List and that of the Concurrent List over the State List. When there is a
conflict between the Concurrent List and the State List, it is the former that should prevail.
Statement d is correct. The power to make laws with respect to residuary subjects is vested in the
Parliament. This residuary power of legislation also includes the power to levy residuary taxes.
Source: Indian Polity by M. Laxmikanth. Chapter 14. Centre-State relation.
Sub) Polity, Indian Constitution, Centre-State relation

Q.49)
Ans) a
Exp) Option a is correct
Bhopal is 23.2599° N.
Jaipur is 26.9124° N
Raipur is 21.2514° N
Kolkata is 22.5726° N.

ForumIAS Offline Guidance Centre


2nd Floor, IAPL House, 19, Pusa Road, Karol Bagh, New Delhi – 110005 | helpdesk@forumias.academy|9821711605
Page 22 of 42

PTS 2021|Simulator Test 3– Solutions|ForumIAS


Bhopal is the nearest city to tropic of cancer (23.5° N) among the given options having latitude of 23.2599° N.
Knowledge Base: The Tropic of Cancer passes through eight states in India:
Gujarat (Jasdan)
Rajasthan (Kalinjarh)
Madhya Pradesh (Shajapur)
Chhattisgarh (Sonhat)
Jharkhand (Lohardaga)
West Bengal (Krishnanagar)
Tripura (Udaipur)---The city closest to the Tropic of Cancer.
Mizoram (Champhai)
Source: Orient Black Swan ATLAS
https://www.outlookindia.com/outlooktraveller/explore/story/69427/facts-about-india-and-the-tropic-
of-cancer
Sub) Geography, Indian Physical Geography, Nearest major cities to tropic of cancer

Q.50)
Ans) a
Exp) Option a is correct.
Statement 1 is correct. OCI card is a life-long visa for entry into and stay in India with a number of other
major benefits attached to it which are not available to other foreigners. A foreigner of Indian origin or a
foreign spouse of an Indian citizen or foreign spouse of an Overseas Citizen of India (OCI) cardholder, can
be registered as an OCI cardholder. OCI cardholders enjoy almost all the privileges of an Indian national
except the right to vote, government service and buying agricultural land.
Statement 2 is incorrect. OCI is a foreign citizen of Indian origin who is allowed to live and work in India
for an indefinite period. NRI is Indian citizen who has been residing in foreign country.
Statement 3 is incorrect. OCI cardholders are neither permitted to vote, nor are they eligible to run for
public office nor hold government jobs in India, among a few other things. Also, like NRIs, OCI
Cardholders are permitted to purchase immovable property in India, but they are not permitted to purchase
agricultural land or a farmhouse or plantation property.
Source: https://economictimes.indiatimes.com/nri/migrate/expert-take-all-your-questions-about-the-
recent-oci-changes-answered-/articleshow/82059519.cms
Sub) Polity, Indian Constitution, Changes in the OCI card

Q.51)
Ans) c
Exp) Option c is correct.
Option a is correct: Sikandar Lodi took keen interest in agriculture. He established a new measure of yard,
called gaz-i-Sikandari and prepared rent rolls (jama), which became the basis for rent rolls later used by Sher
Shah Suri.
Option b is correct: He introduced the practice of audit in order to check the accounts of muqtas and walis
(governors). Mubarak Khan Lodi (Tuji Khail), the governor of Jaunpur, was the first noble whose accounts
came under scrutiny in 1506. He was found guilty of embezzlement and, therefore, dismissed.
Option c is incorrect: Sikandar Lodi laid great emphasis on law and order, and all highways were made safe
from dacoits and robbers.
Option d is correct: He imposed a ban on the use of any language other than Persian in the government
offices. This led many Hindus to learn Persian and they acquired proficiency in Persian within a short span

ForumIAS Offline Guidance Centre


2nd Floor, IAPL House, 19, Pusa Road, Karol Bagh, New Delhi – 110005 | helpdesk@forumias.academy|9821711605
Page 23 of 42

PTS 2021|Simulator Test 3– Solutions|ForumIAS


of time. Consequently, they began to look after and supervise the revenue administration. When Babur
came to India, he was astonished to see that the revenue department was completely manned by the Hindus.
Knowledge Base: The Lodis became the rulers of the Delhi Sultanate by displacing the Sayyid dynasty in
1451. They were of Afghan descent, after a long line of Turkish rulers. Sikandar Lodi was a contemporary of
Mahmud Begarha of Gujarat, and Rana Sanga of Mewar, against both of whom he later fought wars for
territorial supremacy. He tried to control the independence loving Afghan nobles of his courts, however he
got limited success in these matters. He also abolished the octroi duty on grains. He developed Agra as an
important city and later as a second capital of the Lodi dynasty.
Source: E-gyankosh: UNIT 5 EARLY MUGHALS AND AFGHANS.
Sub) History, Medieval History, Sikander Lodhi

Q.52)
Ans) a
Exp) Option a is correct.
Statement 1 is correct: It's a smouldering fire from a previous growing season that can smoulder under the
ground's carbon-rich peat. The fire can rekindle as the weather warms. Holdover fires are another name for
these types of fires.
Statement 2 is correct: The fires and record temperatures had the potential of turning the carbon sink into a
carbon source and increasing global warming, the experts noted. The team tracked fire activity in the
Russian Arctic in real time, using a variety of satellite and remote sensing tools.
Statement 3 is incorrect: The fire regimes in the Arctic are changing rapidly, with ‘zombie fires’ becoming
more frequent in addition to fires occurring in the once-frozen tundra. So, it is most commonly seen in
Arctic region not in Tropical regions.
Source: https://www.downtoearth.org.in/news/climate-change/zombie-fires-in-a-warming-arctic-region-
a-worry-study-73605
Sub) Environment, Sustainable Practices , Zombie Fire

Q.53)
Ans) a
Exp) Option a is correct
Statement 1 is correct. Kharosthi seems to be derived from a form of Aramaic used as a in administrative
work during the reign of Darius the Great. The use of Kharoshti in ancient Indian architecture is the result
of India's contact with Iran. The early inscriptions of northwestern India, of which the oldest are the
Aśokan edicts, are written in the so-called kharosthi script.
Statement 2 is incorrect. The Iranian coins are found in north-west frontier region pointing to the existence
of trade with Iran. However, it is wrong to think that the punch-marked coins continued in India as a result
of contact with Iran. Knowledge Base:
Other influences of Iranian Invasion:
1) Iranian invasion gave an impetus to Indo-Iranian trade and commerce.
2) It has considerable influence on Mauryan sculpture. The monument of Ashokan times, especially the
bell-shaped capitals, derived its architecture from Iranian sculpture.
3) The influence may also be traced in the preamble of Asoka’s edicts as well as in certain words used in
them. For instance, for the Iranian term dipi the Ashoka scribe used the term lipi.
4) Through the Iranians, the Greeks came to know about the great wealth of India and eventually led to the
Alexanders invasion of India.
Source: Ancient History, NCERT XI, Chapter-11, Pg. 76

ForumIAS Offline Guidance Centre


2nd Floor, IAPL House, 19, Pusa Road, Karol Bagh, New Delhi – 110005 | helpdesk@forumias.academy|9821711605
Page 24 of 42

PTS 2021|Simulator Test 3– Solutions|ForumIAS


https://www.iranicaonline.org/articles/asoka-mauryan-emperor#pt2
https://amp.blog.shops-net.com/559437/1/kharosthi.html
Sub) History, Ancient History, Iranian influence

Q.54)
Ans) b
Exp) Option b is correct.
Statement 1 is incorrect. The Convention on the Legal Status of the Caspian Sea was not signed under the
aegis of the United Nations. It was signed in 2018, by Azerbaijan, Iran, Kazakhstan, Russia and
Turkmenistan (all littoral countries of the Caspian Sea).
Statement 2 is correct. The Convention decided to treat the Caspian’s surface as a sea. It gave jurisdiction to
the littoral countries over waters extending up to 15 nautical miles (nm) from their coasts as well as fishing
rights over an additional 10 nm.
Statement 3 is incorrect. The Convention did not reach a settlement on the precise formula for allocating
the sea bed and its mineral deposits. It said that these issues are to be decided by the littoral countries on a
bilateral basis.
Statement 4 is incorrect. The Convention forbids non-littoral countries from deploying military vessels in
the waters of the Caspian. But it permits the construction of pipelines, with the approval of the countries
whose sea beds they transit.
Source: https://iranintl.com/en/iran-in-brief/russia-demands-iran-ratify-caspian-sea-legal-convention
Sub) International Relations, Map Based, Caspian sea+ Caspian sea treaty

Q.55)
Ans) b
Exp) Option b is correct.
Statement 1 is incorrect: The Disinvestment/Strategic Disinvestment Policy identified four sectors --
Atomic energy, Space and Defence; Transport and Telecommunications; Power, Petroleum, Coal and other
minerals; and Banking, Insurance and financial services -- as strategic sectors, where bare minimum CPSEs
would be retained. In strategic sectors, there will be bare minimum presence of the public sector enterprises.
The remaining CPSEs in the strategic sectors will be privatised or merged or subsidiarized with other
CPSEs or closed. In non-strategic sectors, CPSEs will be privatised, otherwise shall be closed.
Statement 2 is correct: NITI Aayog will work out on the list of Central Public Sector companies that would
be taken up for strategic disinvestment.
Source: https://www.livemint.com/news/india/gom-to-decide-psu-count-in-strategic-sectors-dipam-
secretary-11612359970185.html
https://www.business-standard.com/budget/article/fy22-disinvestment-target-at-rs-1-75-trn-financials-a-
strategic-sector-121020100886_1.html
Sub) Economics, Indian Economy, Disinvestment - Targets Trend/Policy

Q.56)
Ans) b
Exp) Option b is correct.
Option a is incorrect. BIMSTEC constitutes seven Member States: five deriving from South Asia, including
Bangladesh, Bhutan, India, Nepal, Sri Lanka, and two from Southeast Asia, including Myanmar and
Thailand.

ForumIAS Offline Guidance Centre


2nd Floor, IAPL House, 19, Pusa Road, Karol Bagh, New Delhi – 110005 | helpdesk@forumias.academy|9821711605
Page 25 of 42

PTS 2021|Simulator Test 3– Solutions|ForumIAS


Option b is correct. Initially, the economic bloc was formed with four Member States with the acronym
‘BIST-EC’ (Bangladesh, India, Sri Lanka and Thailand Economic Cooperation). Following the inclusion of
Myanmar on 22 December 1997, the Group was renamed ‘BIMST-EC’. With the admission of Nepal and
Bhutan at the 6th Ministerial Meeting (2004), the name of the grouping was changed to BIMSTEC.
Option c is incorrect. BIMSTEC has organized 4 summits till now. The fourth summit was held in
Kathmandu, Nepal in 2018.
Option d is incorrect. Permanent Secretariat of BIMSTEC was established in Dhaka, Bangladesh on 13th
September 2014.
Source: https://www.thehindu.com/opinion/op-ed/bimstec-needs-to-reinvent-itself/article34313033.ece
Sub) International Relations, Multilateral Organisations, BIMSTEC

Q.57)
Ans) c
EXP) Option c is correct.
Mission Karmayogi: It is also known as the National Program for Civil Services Capacity Building. The
mission aims to improve the Human Resource management practices in the Government. It will use scale &
state of the art infrastructure to augment the capacity of Civil Servants.
PM Kaushal Vikas Yojana: The objective of this Skill Certification Scheme is to enable a large number of
Indian youth to take up industry-relevant skill training that will help them in securing a better livelihood.
Individuals with prior learning experience or skills will also be assessed and certified under Recognition of
Prior Learning (RPL). The scheme has been launched by the Ministry of Skill development and
entrepreneurship.
Nai Udaan: Ministry of Minority Affairs is implementing the scheme to support the Minority Students for
preparation of Main Examination,
PM Mudra Yojana: The Pradhan Mantri Mudra Yojana or PMMY is a flagship scheme of the Government
of India to extend affordable credit to micro and small enterprises.
Source: https://timesofindia.indiatimes.com/india/nai-udaan-scheme-to-help-civil-services-aspirants-
from-minority-communities-jitendra-singh/articleshow/77619585.cms
Sub) Polity, Indian Constitution, Civil Services reform-Mission Karmayogi

Q.58)
Ans) c
Exp) Option c is correct.
Statement 1 is correct – The Inter Tropical Convergence Zone (ITCZ,) is where the northeast and the
southeast trade winds converge. This convergence zone lies more or less parallel to the equator but moves
north or south with the apparent movement of the sun. It is a broad trough of low pressure in equatorial
latitudes. In the month of July, because of intense heating of Indian peninsula, the ITCZ shifts northwards,
roughly parallel to Himalayas between 20°-25° N latitude.
Statement 2 is incorrect – The shift in the position of the ITCZ is related to the phenomenon of the
withdrawal of the westerly jet stream from its position over the north Indian plain, south of the Himalayas.
Once the Westerly jet stream has withdrawn, the easterly Jet Stream (Somali Jet) sets in along 15°N latitude
only after the western jet stream has withdrawn itself from the region. This easterly jet stream is held
responsible for the burst of the monsoon in India.
Statement 3 is correct – The ITCZ being a zone of low pressure, attracts inflow of winds from different
directions. The maritime tropical air mass from the southern hemisphere, after crossing the equator, rushes
to the low-pressure area in the general south-westerly direction. It is this moist air current which is

ForumIAS Offline Guidance Centre


2nd Floor, IAPL House, 19, Pusa Road, Karol Bagh, New Delhi – 110005 | helpdesk@forumias.academy|9821711605
Page 26 of 42

PTS 2021|Simulator Test 3– Solutions|ForumIAS


popularly known as the southwest monsoon. Source: NCERT Class XI – India: Physical Environment,
Chapter 4, page 36
Sub) Geography, Indian Physical Geography, Role of ITCZ in Monsoon

Q.59)
Ans) b
Exp) option b is correct.
‘Eco-Sensitive Zones’ (ESZ),:
Option a is incorrect. Eco-Sensitive Zones (ESZs) or Ecologically Fragile Areas (EFAs) are areas around
Protected Areas, National Parks, and Wildlife Sanctuaries in India that have been notified by the Ministry
of Environment, Forests, and Climate Change (MoEFCC).
ESZs are notified by MoEFCC, Government of India under Environment Protection Act 1986.
Option b is correct According to the Wildlife Conservation Strategy of 2002, an ESZ could extend up to 10
kilometers around a protected area. Furthermore, sensitive pathways, connectivity, and ecologically
significant patches that are critical for landscape linkage should be included in the Eco-Sensitive Zones if
their width exceeds 10 kilometers.
Declaring ESZs has the aim of acting as "shock absorbers" for protected areas by controlling and
maintaining activities in and around them. They often serve as a buffer zone between areas with high
protection and those with lower protection.
Option c and d are incorrect. The Central Government has the authority to limit the areas in which any
industry, activity, or process, or class of industries, operations, or processes, shall not be carried out or shall
be carried out with certain safeguards.
Source: https://www.conservationindia.org/resources/moef-releases-new-guidelines-for-eco-sensitive-
zones-eszs-around-protected-areas
Sub) Environment, Ecosystem, Ecologically Sensitive Area

Q.60)
Ans) b
Exp) Option b is correct.
Statement 1 is correct. The provision of providing Central wetland regulatory authority (CWRA) at the
central level has been omitted in 2017 rules and has been replaced by a national committee. 2017 rules
provide for the establishment of state wetland authorities at the state level and in UT.
Statement 2 is incorrect. According to the 2010 rules, anyone aggrieved with the CWRA’s decisions could
have filed an appeal with the National Green Tribunal, but the new 2017 rules are silent on the appeal
process.
Statement 3 is correct. The following wetlands have been omitted from the definition of wetlands
(a) River channels; (b) Paddy fields; (c) Human-made waterbodies specifically constructed for drinking
water purposes; (d) Human-made waterbodies specifically constructed for aquaculture purposes; (e)
Human-made waterbodies specifically constructed for salt production purposes; (f) Human-made
waterbodies specifically constructed for recreation purposes; (g) Human-made waterbodies specifically
constructed for irrigation purposes; (h) Wetlands falling within areas covered under the Indian Forest Act,
1927; Forest (Conservation) Act, 1980; State Forest Acts and amendments thereof; (i) Wetlands falling
within areas covered under the Wildlife (Protection) Act, 1972 and amendments thereof; (j) Wetlands
falling within areas covered under the Coastal Regulation Zone Notification, 2011 and amendments thereof
Source: page-4 of http://moef.gov.in/wp-content/uploads/2020/01/final-version-and-printed-wetland-
guidelines-rules-2017-03.01.20.pdf;

ForumIAS Offline Guidance Centre


2nd Floor, IAPL House, 19, Pusa Road, Karol Bagh, New Delhi – 110005 | helpdesk@forumias.academy|9821711605
Page 27 of 42

PTS 2021|Simulator Test 3– Solutions|ForumIAS


https://www.livemint.com/Politics/y6Tr3tkrr3q28AmGKaBFII/Environment-ministry-notifies-new-
wetland-rules.html
Sub) Environment, Convention / Organisation/ Laws/ Protocols, wetlands management rules - recent
changes

Q.61)
Ans) a
Exp) Option (a) is the correct answer.
Out of the four statements, term deposits are the least liquid as the depositor can withdraw money after a
specific period. Currency and coins with the public are the most liquid as they can be spent very easily.
Demand deposits will have more liquidity than the saving deposits as the former includes the ‘current
accounts’ of the firms (the most liquid deposit with the banks).
Source) Indian Economy; Ramesh Singh- Ch-12 Banking in India
Sub) Economics, Social Development, North East Venture Fund

Q.62)
Ans) d
Exp) Option d is correct.
Option d is correct. Dr. B. R Ambedkar was appointed by the Bombay Legislative Council to work with the
Simon Commission. He argued for universal adult franchise for both males and females alike; for
provincial autonomy in the provinces and dyarchy at the centre. On behalf of the Bahishkrita Hitakarini
Sabha, he submitted a memorandum on the rights and safeguards required for the depressed classes. He
demanded political protection of the depressed classes and representation on the same basis as the
Mohammedan minority. Source: Modern History, Spectrum, 6th Edition, Chapter-18, Pg. 359
Sub) History, Modern History, Ambedkar and Simon commission

Q.63)
Ans) d
Exp) Option (d) is the correct answer.
Statement 1 is incorrect. Outcome budget is presented by different departments and divisions of a ministry
or the government, the Performance budget is presented by the Ministry of Finance on behalf of the
government. Both go for ‘quantitative’ as well as ‘qualitative’ progress reports of the performance.
Statement 2 is correct. The outcome budget is a micro level process because it is related to separate
departments and ministries while performance budget is a macro-level process in budgeting involving the
whole government machinery.
Statement 3 is correct. There are many outcome budgets in any one performance budget. The basic objective
of such budgeting is to bring in transparency and thereby making the government more and more
responsible to the House and the public. Naturally, they bring in prudence and optimisation elements in
public spending.
Source) Indian Economy – Ramesh Singh, Chapter 18 (Public Finance in India), Page No. 18.25
Sub) Economics, Schemes, Ease of doing business ranking of Indian states.

Q.64)
Ans) a
Exp) Option a is correct

ForumIAS Offline Guidance Centre


2nd Floor, IAPL House, 19, Pusa Road, Karol Bagh, New Delhi – 110005 | helpdesk@forumias.academy|9821711605
Page 28 of 42

PTS 2021|Simulator Test 3– Solutions|ForumIAS


The Great Reset is an initiative by the World Economic Forum. It is based on the assessment that the world
economy is in deep trouble. Executive chairman of the WEF Schwab has argued that the situation has been
made a lot worse by many factors, including the pandemic’s devastating effects on global society, the un-
folding technological revolution, and the consequences of climate change.
Schwab recommended that “the world must act jointly and swiftly to revamp all aspects of our societies and
economies, from education to social contracts and working conditions. Every country, from the United
States to China, must participate, and every industry, from oil and gas to tech, must be transformed. In
short, we need a ‘Great Reset’ of capitalism.
Source: https://indianexpress.com/article/explained/what-is-the-great-reset-and-why-is-it-controversial-
world-economic-forum-7160434/
Sub) Economics, Schemes, The Great reset initiative

Q.65)
Ans) d
Exp) Option d is correct.
Statement 1 is incorrect. Every occupier, i.e. a person having administrative control over the institution and
the premises generating biomedical waste shall pre-treat the laboratory waste, microbiological waste, blood
samples, and blood bags through disinfection or sterilization on-site in the manner as prescribed by the
World Health Organization (WHO) or guidelines on safe management of wastes from health care activities
and WHO Blue Book 2014 and then sent to the Common bio-medical waste treatment facility for final
disposal.
Statement 2 is incorrect. The land for setting up of common bio-medical waste treatment facility is
provided by state government.
Source: Shankar IAS Chapter-25v ACTS and Policies page-307
Sub) Environment, Convention / Organisation/ Laws/ Protocols, Natural Capital Accounting and
Valuation of Ecosystem Services (NCAVES)

Q.66)
Ans) a
Exp) Option a is correct.
Statement 1 is incorrect. The snowline in eastern Himalayas is at a higher altitude due to the lower height of
the eastern Himalayas and the moderating influence of the South west monsoon winds.
Statement 2 is correct. The average height of western Himalayas is higher compared to eastern Himalayas as
in case of western Himalayas. The Indo plate is pushing towards the Eurasian plate and causing the height to
increase more rapidly.
Statement 3 is incorrect. Eastern Himalayas have same orogeny as western Himalayas however their scale of
relief and morphology is different.
Source) Geography of India by Majid Hussain, Chapter 2, Physiography, Page 2.18-2.20
India a comprehensive geography by D.R. Khullar. Page 63 -66
Sub) Geography, Indian Physical Geography, Eastern Himalayas

Q.67)
Ans) d
Exp) Option d is correct.
Option a, b and are correct. Himalayan brown bear- It can be found in some of the most remote mountain
regions of Nepal, India, Pakistan and Tibet and also in parts of south and western Ladakh, and prefers high-

ForumIAS Offline Guidance Centre


2nd Floor, IAPL House, 19, Pusa Road, Karol Bagh, New Delhi – 110005 | helpdesk@forumias.academy|9821711605
Page 29 of 42

PTS 2021|Simulator Test 3– Solutions|ForumIAS


altitude, open valley. It is listed under Appendix I in CITES and Schedule I of the Wildlife (Protection) Act,
1972. Shrinking of breeding ground has forced the bears to reach lower areas and human settlements in
search of food. Another possible reason behind the rise of such conflicts is believed to be climate change.
Option d is incorrect. Shorter winters and long summers could have had a disturbing effect on their
hibernation cycles.
Knowledge Base: Himalayan brown bear is critically endangered as per IUCN red list.
Snow leopard do not hibernate but live throughout the entire year with no long period of sleep. It is also
listed under Appendix I in CITES and Schedule I of the Wildlife (Protection) Act, 1972. It is listed as
vulnerable as per IUCN list
Sloth bear lives in wet and dry tropical forests, grasslands. It is a lowland species. It is also listed under
Appendix I in CITES and Schedule I of the Wildlife (Protection) Act, 1972.
Hangul is native to India. It is listed as critically endangered as per IUCN Red list.
Source: https://lifestyle.livemint.com/smart-living/environment/species-in-focus-the-himalayan-brown-
bear-faces-the-brunt-of-man-animal-conflict-111602359403603.html
Sub) Environment, Biodiversity, Himalayan Brown Bear

Q.68)
Ans) b
Exp) Option b is correct.
Statement 1 is incorrect. Non-Aligned Movement was founded at the First Summit Conference of Belgrade,
which was held on September, 1961. The Bandung Asian-African Conference (1955) is considered as the
most immediate antecedent to the creation of the Non-Aligned Movement.
Statement 2 is correct. As a condition for membership, the states of the Non-Aligned Movement cannot be
part of a multilateral military alliance such as NATO. It promotes independence of non-aligned countries
from great power or block influences and rivalries.
Source: https://www.thehindu.com/opinion/op-ed/what-is-in-a-nam-and-indias-
alignment/article32555378.ece
Sub) International Relations, Multilateral Organisations, NAM

Q.69)
Ans) a
Exp) Option a is correct
Option a is correct. The focus of these Mughal era miniature paintings was to glorify the rulers and show
their life and household, rather than depicting the god. They focused on royal pomp and show, hunting
scenes, the battles, historical events and other court related paintings. The Mughal paintings brought
together Persian naturalistic style with the opulence of a great dynasty and created some beautifully
illustrated folios.
Option b is incorrect. The use of 3 dimensional figures and the continued use of foreshortening was the
defining features of paintings in Akbar’s period (not Shah Jahan). During Akbar’s reign, the artists
encouraged the use of calligraphy in the paintings. One of the distinguishing features of this period was the
transformation of popular art to the court art, i.e., the artist was more focused on depicting the scenes of
court life than the life of the masses.
Option c is incorrect. The Mughal paintings reached its zenith in the period of Jahangir. He was a naturalist
by nature and preferred the paintings of flora and fauna, i.e. birds, animals, trees and flowers. He shifted
from illustrated manuscripts to album and emphasised on bringing naturalism to portrait (individual)

ForumIAS Offline Guidance Centre


2nd Floor, IAPL House, 19, Pusa Road, Karol Bagh, New Delhi – 110005 | helpdesk@forumias.academy|9821711605
Page 30 of 42

PTS 2021|Simulator Test 3– Solutions|ForumIAS


painting. One of the unique trends that developed in this period was of decorated margins around the
paintings that were sometimes as elaborate as the paintings themselves.
Option d is incorrect. The Thanjavur or Tanjore School of miniature paintings (and not Mughal era) are
unique because of the use of brilliant colour patterns and the liberal use of gold leaf. They used many types
of gemstones and cut glasses for embellishments to create larger than life images. They are mostly created
on glass and wooden planks (Palagai padam) instead of cloth and vellum as preferred in North India. It has
been recognised as Geographical Indication (GI) by Government of India.
Knowledge Base:
1) Mughal era paintings brought the technique of foreshortening to the Indian painter’s repertoire. Under
this technique, “objects were drawn in a way that they look closer and smaller than they really are”.
2) Shah Jahan liked creating artificial elements in the paintings, instead of naturalistic depictions. It is said
that he tried to reduce the liveliness of the paintings and bring in unnatural stillness as he was inspired
by the European influence. He also ordered to increase the use of gold and silver in the paintings.
Source: Indian Art & Culture, Nitin Singhania, 3rd Edition, Chapter-2
Sub) Art and Culture, Sculpture & Paintings, Mughal era Miniature paintings

Q.70)
Ans) d
Exp) Option d is correct.
Pair 1 is incorrectly matched. Kundhei are the string puppets (and not shadow puppets) of Odisha. They are
made of light wood and are dressed in long skirts. The puppets have more joints, thus giving the puppeteer
more flexibility. The strings are attached to a triangular prop. There is a marked influence of Odissi dance in
Kundhei puppet shows.
Pair 2 is correctly matched. Gombeyatta is the traditional string puppet show of Karnataka. They are styled
and designed based on the various characters of the Yakshagana Theatres. A unique feature of this puppetry
is the use of more than one puppeteer to manipulate the puppets.
Pair 3 is correctly matched. Pavakoothu is the traditional glove puppet show of Kerala. It originated around
18th century AD. The puppets are decorated with colourful headgears, feathers and face paints which is an
evidence of a heavy influence of Kathakali dance form. The plays are themed around narrations of
Ramayana and Mahabharata. Source: Indian Art & Culture, Nitin Singhania, 3rd Edition, Chapter-8
Sub) Art and Culture, Miscellanoeus - Martial Arts / Calendar / Festivals, Puppetry

Q.71)
Ans) d
Exp) Option d is correct.
Statement 1 is correct. The length of the frost-free season (and the corresponding growing season) has been
increasing since the 1980s, affecting ecosystems and agriculture. Across the world, the growing season is
projected to continue to lengthen.
Statement 2 is correct. The intensity of cyclones, hurricanes and typhoons might increase in the next
century due to global warming, a new joint British-American study by National Oceanic and Atmospheric
Administration (NOAA) and Princeton University in the United States and University of East Anglia in the
United Kingdom has said.
Statement 3 is correct. Because human-induced warming is superimposed on a naturally varying climate,
the temperature rise has not been, and will not be, uniform or smooth across the country or over time.
Global temperatures increased by over 1oF during the past century and are projected to increase 2.5-10.4oF
by 2100 as a result of human emissions of greenhouse gases.

ForumIAS Offline Guidance Centre


2nd Floor, IAPL House, 19, Pusa Road, Karol Bagh, New Delhi – 110005 | helpdesk@forumias.academy|9821711605
Page 31 of 42

PTS 2021|Simulator Test 3– Solutions|ForumIAS


Source: https://science.thewire.in/environment/will-the-new-farm-laws-lead-to-more-greenhouse-gas-
emissions-from-indias-farms/
https://climate.nasa.gov/effects/#:~:text=Global%20climate%20change%20has%20already,and%20trees%2
0are%20flowering%20sooner
Sub) Environment, Climate Change, observable effects of climate change

Q.72)
Ans) b
Exp) Option b is correct.
Statement 1 is correct. Reflation is a fiscal or monetary policy enacted after a period of economic slowdown
or contraction. Here the goal is to expand output, stimulate spending and curb the effects of deflation.
Reflation policies can include reducing taxes, changing the money supply and lowering interest rates.
Statement 2 is incorrect. Disinflation is slower rate of inflation. It means that there is still rise in prices but
overall rate is slow. It is simply a slowing of Inflation.
Statement 3 is incorrect. Deflation is a decrease in the general price levels of goods and services. It is
opposite of Inflation. During deflation prices of goods and services tend to fall.
Statement 4 is correct. Skewflation means that some sectors are facing inflation while other sectors of
economy do not.
Source: Ramesh Singh’s “Indian Economy”. Chapter –7
Sub) Economics, Schemes, Emergency Credit Line Guarantee Scheme

Q.73)
Ans) d
Exp) Option d is correct.
The Chemical structure of DNA and RNA is a polynucleotide chain. A nucleotide has three components – a
nitrogenous base, a pentose sugar (ribose in case of RNA, and deoxyribose for DNA), and a phosphate
group. There are two types of nitrogenous bases – Purines (Adenine and Guanine), and Pyrimidines
(Cytosine, Uracil and Thymine).
Purines i.e. Adenine and Guanine are present in both DNA and RNA.
Pyrimidines - Thymine is present in DNA while Uracil is present in RNA. Cytosine is common for both
DNA and RNA. Source: NCERT Biology class XII – Chapter 6, Page 96-97
Sub) Science and Technology, Biology, DNA present in the body – basic

Q.74)
Ans) c
Exp) Option c is correct.
Option 1 is correct. Solar Panels are durable and can perform under high temperatures. But same as other
electronics, with higher temperatures, come with decreased power outputs. The efficiency of a solar panel
tends to decrease as the temperatures rise, which can be concerning for property owners who are looking to
get a solar panel installation in a hot and sunny climate. The bodies of water that host the floating solar
panels will help the solar-powered systems cool down, which means that the floating solar panel installation
will increase the efficiency of a solar panel in the hot climates than they otherwise would.
Option 2 is incorrect. A floating solar panel installation requires additional costs than more traditional solar
panel installations. Because this is a relatively new solar power technology it requires specialized solar power
equipment and more niche solar panel installation knowledge, it will typically require a higher price tag than
installing a similar-sized solar power farm on a rooftop, or solid ground.

ForumIAS Offline Guidance Centre


2nd Floor, IAPL House, 19, Pusa Road, Karol Bagh, New Delhi – 110005 | helpdesk@forumias.academy|9821711605
Page 32 of 42

PTS 2021|Simulator Test 3– Solutions|ForumIAS


Option 3 is correct. The floating solar panel installation provides shade to the body of water and reduces the
evaporation from these ponds, reservoirs, and the lakes. This is a particularly useful environmental benefit
of solar energy in areas that are more susceptible to droughts, as water loss due to evaporation and can add
up over time and contribute to a shortage.
Knowledge Base: The world's largest floating 600 MW solar energy project to be constructed at
Omkareshwar dam on Narmada river in Khandwa district of Madhya Pradesh will begin power generation
by year 2022-23.
Source: http://www.suniboat.com/pros-and-cons-of-solar-on-
water.html#:~:text=One%20of%20the%20major%20advantages,of%20solar%20energy%20into%20electrici
ty.
https://energy.economictimes.indiatimes.com/news/renewable/worlds-largest-floating-solar-project-to-
start-in-mp-by-2023-state-govt/80125623
Sub) Science and Technology, Energy, nuclear reactors - Thiruvanthapuram issue

Q.75)
Ans) c
Exp) Option c is correct.
Statement 1 is incorrect. The oldest Jain pilgrimage sites are to be found in Bihar (and not in Karnataka).
Many of these sites are also famous for early Buddhist shrines. Jains were prolific temple builders like the
Hindus, and their sacred shrines and pilgrimage spots are to be found across the length and breadth of India
except in the hills.
Statement 2 is incorrect. The port-town of Nagapattinam was a major Buddhist centre right until the Chola
Period. One of the reasons for this must have been its importance in trade with Sri Lanka where large
numbers of Buddhists still live. Bronze and stone sculptures in Chola style have come to light at
Nagapattinam and generally date back to the tenth century.
Statement 3 is correct. The Jain temples at Mount Abu were constructed by Vimal Shah. Notable for a
simplistic exterior in contrast with the exuberant marble interiors, their rich sculptural decoration with
deep undercutting creates a lace-like appearance. The temple is famous for its unique patterns on every
ceiling, and the graceful bracket figures along the domed ceilings.
Knowledge Base: Other important Jain temples:
In central India, Deogarh, Khajuraho, Chanderi and Gwalior have some excellent examples of Jain temples.
1) In the Deccan, some of the most architecturally important Jain sites can be found in Ellora and Aihole.
2) Karnataka has a rich heritage of Jain shrines and at Sravana Belagola the famous statue of Gomateshwara,
the granite statue of Lord Bahubali which stands eighteen metres or fifty-seven feet high, is the world’s
tallest monolithic free-standing structure.
3) Palitana temples, Shatrunjay hills, Gujarat is regarded as the holiest site, especially by Svetambara sect.
4) It is believed that 20 tirthankars attained Moksha at Shikarji, Parasnath in Jharkhand.
5) The last tirthankar Mahavira was cremated at Pawapuri, Nalanda district in Bihar.
6) Derasar is a word used for Jain temples in Gujarat and Southern Rajasthan. In Karnataka, Jain temples are
known as Basadis.
Source: Indian Art & Culture, NCERT, Chapter-6, Pg. 94-95
Indian Art & Culture, Nitin Singhania, 3rd Edition, Chapter-1
Sub) Art and Culture, Architecture, Jain temples

ForumIAS Offline Guidance Centre


2nd Floor, IAPL House, 19, Pusa Road, Karol Bagh, New Delhi – 110005 | helpdesk@forumias.academy|9821711605
Page 33 of 42

PTS 2021|Simulator Test 3– Solutions|ForumIAS


Q.76)
Ans) b
Exp) Option b is correct.
Under Sixth Schedule of the Constitution, the governor of the state is empowered to direct that an act of the
legislature of the state does not apply to a tribal area in the state or apply with specified modifications and
exceptions.
The Governor of Assam may likewise direct that an act of Parliament does not apply to a tribal area in the
state or apply with specified modifications and exceptions. The President enjoys the same power with
respect to tribal areas in Meghalaya, Tripura and Mizoram.
Source: Indian Polity by M. Laxmikanth. Chapter 37. Scheduled and Tribal area.
Sub) Polity, Indian Constitution, Power of Governor in 6th schedule areas

Q.77)
Ans) c
Exp) Option c is correct.
Statement 1 is incorrect: Generally, there are two parties in an insurance transaction – the insured (first
party) and the insurer (second party). In third-party insurance, the first party seeks protection from claims
of losses and damages by a third party.
This can be elaborated with an example. Consider the owner of a car. He is the first party. Suppose he has
hit a pedestrian on the road. Now the pedestrian can initiate liability charges against the first party. Hence,
the pedestrian, the beneficiary, is the third party. The insurance company will accordingly pay the damages
to the third party on behalf of the first party. The insurance company is the second party.
Hence, in a third-party insurance scheme, the first party is held responsible for losses and damages, while
the third party is the beneficiary. Accordingly, the first party seeks an insurance cover from the insurance
company to cover for any future claims of losses and damages.
Statement 2 is correct: Automobile insurance is one of the most common types of third-party insurance.
Statement 3 is correct: IRDAI has recently withdrawn the requirement for compulsory long-term third-
party cover for four and two-wheelers. This is going to bring down the costs of buying automobiles by a
significant margin.
Source: https://indianexpress.com/article/business/irdai-withdraws-long-term-third-party-motor-
insurance-for-3-5-years-from-aug-1-6451116/
https://economictimes.indiatimes.com/definition/third-party-insurance
Sub) Economics, Indian Economy, Bharat Interface for Money-Unified Payments Interface

Q.78)
Ans) b
Exp) Option b is correct.
Option a is incorrect. Molecular Diagnosis: It is the use of molecular biology for early detection of diseases
(at a time when the symptoms of the disease are not yet visible). Here, very low concentration of a bacterial
or viral disease-causing agent can be detected by amplification of their nucleic acid using techniques like
Polymerase Chain Reaction (PCR) and Enzyme Linked Immuno-Sorbent Assay (ELISA).
Option b is correct. Gene Therapy: It is the insertion of genes into an individual’s cells and tissues to treat
diseases especially hereditary diseases. It does so by replacing a defective mutant allele with a functional one
or gene targeting which involves gene amplification.
Option c is incorrect. Active immunity: A type of acquired immunity whereby resistance to a disease is built
up by either having the disease or receiving a vaccine against it.

ForumIAS Offline Guidance Centre


2nd Floor, IAPL House, 19, Pusa Road, Karol Bagh, New Delhi – 110005 | helpdesk@forumias.academy|9821711605
Page 34 of 42

PTS 2021|Simulator Test 3– Solutions|ForumIAS


Option d is incorrect. Crossbreeding: Interbreeding to breed (animals or plants) using parents of different
races, varieties, breeds, etc.
Source: NCERT Biology Class XII, Chapter-12, Page 211-212
Sub) Science and Technology, Biotechnology, Application of Biotechnology in medicines e.g.
Pharmacogenomics

Q.79)
Ans) c
Exp) Option (c) is the correct answer.
Physical capital implies the non-human assets of the company, such as plant and machinery, tools and
equipment, office supplies etc. that help in the process of production. Human capital refers to the stock of
knowledge, talent, skills and abilities brought in by the employee to the organization.
Option a is incorrect. Human capital is intangible and Physical capital is tangible.
Option b is incorrect. Human capital is not sold in the market whereas Physical capital is. Option c is
correct. Physical capital is completely mobile across countries, but Human capital is not.
Option d is incorrect. Both forms of capital depreciate over a period of time but the nature of depreciation
differs between the two. Continuous use of machine leads to depreciation along with change in technology
makes a machine obsolete. In the case of human capital, depreciation takes place with ageing but can be
reduced, to a large extent, through continuous investment in education, health, etc.
Source) Chapter 5, Indian Economic Development NCERT XI
Sub) Economics, Indian Economy, PPP concept

Q.80)
Ans) c
Exp) Option c is correct.
Option a is incorrect. Of the nine migratory bird flyways, the Central Asian flyway goes from India.
Option b is incorrect. Kalrav is the 1st bird festival of Bihar organised recently at Nagi-Nakti bird sanctuaries
in Jamui district.
Option c is correct. The State of India’s Bird 2020 was the first-of-its-kind analysis released at the 13th
Conference of Parties of the Convention on the Conservation of Migratory Species of Wild Animals.
Option d is incorrect. The annual Flamingo Festival is held in Pulicat lake. The Pulicat lake supports rich
biodiversity and high biomass of fishes and planktons which is utilised as food resources by visiting birds.
Thus about 75 aquatic and terrestrial bird species visit the sanctuary every year.
Source: https://www.downtoearth.org.in/news/wildlife-biodiversity/javadekar-hints-at-key-decisions-
on-central-asian-migratory-bird-flyway-69228
https://www.stateofindiasbirds.in/
https://www.hindustantimes.com/lifestyle/festivals/3day-state-bird-festival-to-be-held-from-jan-15-in-
bihar-s-jamui-district-101610279358179.html
Sub) Environment, Biodiversity, ‘Kalrav’ bird festival Nagi-Nakti bird sanctuaries

Q.81)
Ans) b
Exp) Option b is correct.
Statement 1 is correct. Indian Banks' Association (IBA) was formed in 1946. It is a representative body of
management of banking in India operating in India - an association of Indian banks and financial
institutions based in Mumbai.

ForumIAS Offline Guidance Centre


2nd Floor, IAPL House, 19, Pusa Road, Karol Bagh, New Delhi – 110005 | helpdesk@forumias.academy|9821711605
Page 35 of 42

PTS 2021|Simulator Test 3– Solutions|ForumIAS


Statement 2 is correct. The members comprise of: Public Sector Banks, Private Sector Banks, Foreign Banks
having offices in India, Co-operative Banks, Regional Rural Banks and All India Financial Institution.
Statement 3 is incorrect. Those Foreign Banks having office in India can become member of Indian Banks
Association. Source: https://www.iba.org.in/department/payment-systems-and-banking-technology-
2/contact.html
Sub) Economics, Indian Economy, Indian Banks’ Association

Q.82)
Ans) b
Exp) Option b is correct.
Pair 1-D is correctly matched. The Treaty for the Prohibition of Nuclear Weapons in Latin America and the
Caribbean, also known as Treaty of Tlatelolco, was opened for signature on 14 February 1967 in Mexico
City. Its zone of application includes areas of the high seas and the national territories of the 33 countries of
Latin America and the Caribbean.
Pair 2-C is correctly matched. The Treaty of Rarotonga, the South Pacific Nuclear Free Zone Treaty,
opened for signature on 6 August 1985 and entered into force on 11 December 1986. The geographic scope of
the Rarotonga Treaty is vast, extending from the West coast of Australia to the boundary of the Latin
American NWFZ in the east, and from the equator to 60 degrees south, where it meets the boundary of the
zone established by the Antarctic treaty.
Pair 3-A is correctly matched. The Treaty on the Southeast Asia Nuclear Weapon-Free Zone, also known as
the SEANWFZ Treaty or Bangkok Treaty, was signed on 15 December 1995 by ten Southeast Asian States.
The Treaty, which entered into force on 27 March 1997, was registered with the United Nations on 27 June
1997. The Treaty is a key legal instrument in supporting the purposes of ASEAN, which, as articulated in the
ASEAN Charter, include preserving Southeast Asia as a Nuclear Weapon-Free Zone and free from all other
weapons of mass destruction.
Pair 4-B is correctly matched. The African Nuclear-Weapon-Free Zone Treaty, also known as the
‘Pelindaba Treaty’, established the nuclear-weapon-free zone on the African continent. It opened for
signature on 12 April 1996 in Cairo, Egypt and entered into force on 15 July 2009.
Source: https://www.business-standard.com/article/international/arab-league-hails-passing-of-treaty-on-
prohibition-of-nuclear-weapons-121012300306_1.html
Sub) International Relations, Bilateral, Nuclear free zone

Q.83)
Ans) c
Exp) Option c is correct.
Statement 1 is correct. The Conference on Disarmament (CD), was recognized by the Tenth Special Session
on Disarmament of the United Nations General Assembly (SSOD-I) (1978) as a single multilateral
disarmament negotiating forum of the international community. It primarily focuses on cessation of the
nuclear arms race and nuclear disarmament, prevention of an arms race in outer space, weapons of mass
destruction and new systems of weapons like radiological weapons.
Statement 2 is incorrect. The Treaty on the Prohibition of Nuclear Weapons was not finalized under the
Conference of Disarmament. It was approved by the United Nations General Assembly in 2017 and came
into force in 2021. The treaty is not supported by nine countries known to possess nuclear weapons as well
as the NATO alliance.

ForumIAS Offline Guidance Centre


2nd Floor, IAPL House, 19, Pusa Road, Karol Bagh, New Delhi – 110005 | helpdesk@forumias.academy|9821711605
Page 36 of 42

PTS 2021|Simulator Test 3– Solutions|ForumIAS


Statement 3 is correct. The Conference of Disarmament reports to the United Nations General Assembly
annually. The Director-General of the United Nations Office at Geneva is the Secretary-General of the
Conference on Disarmament.
Source: https://www.aninews.in/news/world/asia/india-at-disarmament-conference-says-committed-to-
no-first-use-of-nukes-supports-negotiations-on-fmct20210222191136/
Sub) International Relations, Multilateral Organisations, Conference on Disarmament

Q.84)
Ans) a
Exp) Option a is correct.
1) Pair 1-D is correctly matched. The 11th edition of Indo-US joint special forces exercise Vajra Prahar 2021
was conducted at Special Forces training school located at Bakloh, in Himachal Pradesh. The exercise is
aimed at sharing best practices and experiences in areas such as joint mission planning and operational
tactics.
2) Pair 2-C is correctly matched. SAMPRITI is a joint military exercise between armies of India and
Bangladesh. The exercise is aimed at strengthening the military relations between the two countries, which
allowed the two armies to understand each other's tactical drills and operating techniques.
3) Pair 3-B is correctly matched. Desert Knight is an exercise between Indian Air Force and French Air and
Space Force. It was held in Jodhpur in January, 2021. The exercise marks an important milestone as it
includes fielding of Rafale aircraft by both sides and is indicative of the growing interaction between the
two premier Air Forces.
4) Pair 4-A is correctly matched. The SIMBEX series of exercises between Indian Navy and Republic of
Singapore Navy being conducted annually since 1994, are aimed at enhancing mutual inter-operability and
imbibing best practices from each other. The 2020 exercise was held in Andaman sea.
Source: https://theprint.in/defence/india-withdraws-from-russia-military-exercise-participation-of-
china-pakistan-a-factor/491814/
Sub) International Relations, Map Based, Defence exercise match.

Q.85)
Ans) b
Exp) Option b is correct.
Statement 1 is incorrect. RFID tag uses a small electronic chip which is surrounded by antenna unlike
barcoding which uses a pattern of black bars and white spaces, in which the information is coded.
Statement 2 is incorrect. A tag can be read from up to several feet away and does not, need to be within
direct line-of-sight of the reader to be tracked.
Statement 3 is correct. Radio Frequency Identification (RFID) technology is at the core of FASTag, the
flagship initiative of the Ministry of Road Transport and Highways (MoRTH) for implementing electronic
toll collection across the national highways in India.
Knowledge Base: Radio-Frequency Identification (RFID) uses radio waves to read and capture information
stored on a tag attached to an object. A RFID system is made up of two parts: a tag or label and a reader.
RFID tags or labels are embedded with a transmitter and a receiver. The RFID component on the tags have
two parts: a microchip that stores and processes information, and an antenna to receive and transmit a
signal. The tag contains the specific serial number for one specific object. To read the information encoded
on a tag, a two-way radio transmitter-receiver called an interrogator or reader emits a signal to the tag using
an antenna. The tag responds with the information written in its memory bank.
About FASTag.

ForumIAS Offline Guidance Centre


2nd Floor, IAPL House, 19, Pusa Road, Karol Bagh, New Delhi – 110005 | helpdesk@forumias.academy|9821711605
Page 37 of 42

PTS 2021|Simulator Test 3– Solutions|ForumIAS


FASTag is an electronic toll collection system in India, operated by the National Highway Authority of India
(NHAI). It employs Radio Frequency Identification (RFID) technology for making toll payments directly
from the prepaid or savings account linked to it or directly toll owner.
Source: https://www.financialexpress.com/auto/car-news/explained-rfid-technology-what-role-does-it-
play-in-fastag-price-banks/1793347/
https://www.epc-rfid.info/rfid
Sub) Science and Technology, IT, Telecom related Technologies e.g. NFC, RFID, LTE

Q.86)
Ans) b
Exp) Option b is correct.
Statement 1 is incorrect. The Constitution of India has mentioned the term minority only on two occasions
in Article 29 and in Article 30. But it nowhere defines the term minority. It was left at the discretion of
central government to determine which community would be considered as minority community. The
National Commission for Minority Act, 1992, says that central government will notify who are minorities in
the country. The constitution recognizes linguistic and religious minority.
Statement 2 is correct. The minorities are determined based on the population of a community at the
national level. As a result, Muslims, Sikhs, Buddhist, Jains, Parsis and Christians were declared as
minorities. The Hindus are minority in Jammu and Kashmir and some North Eastern states but were not
declared as a minority as it was done on national level.
Statement 3 is incorrect. National Commission for Minority education act, 2004 identifies minorities at the
national level and it identified the six communities (Muslim, Christian, Sikh, Buddhist, Parsi and Jain
community) as religious minority in India.
Source: http://www.legalserviceindia.com/legal/article-720-who-are-minorities.html
Sub) Polity, Indian Constitution, Definition of minorities

Q.87)
Ans) b
Exp) Option b is correct.
Option a is incorrect. Solar Flares. The magnetic field lines near sunspots often tangle, cross, and
reorganize. This can cause a sudden explosion of energy called a solar flare. Solar flares release a lot of
radiation into space. If a solar flare is very intense, the radiation it releases can interfere with our radio
communications here on Earth.
Option b is correct. Sunspots are areas that appear dark on the surface of the Sun. They appear dark because
they are cooler than other parts of the Sun’s surface. The temperature of a sunspot is still very hot though—
around 6,500 degrees Fahrenheit. They are relatively cool because they form at areas where magnetic fields
are particularly strong. These magnetic fields are so strong that they keep some of the heat within the Sun
from reaching the surface.
Option c is incorrect. A prominence is a large, bright, gaseous feature extending outward from the Sun's
surface, often in a loop shape. Prominences are anchored to the Sun's surface in the photosphere, and
extend outwards into the solar corona.
Option d is incorrect. The solar cycle is the cycle that the Sun's magnetic field goes through approximately
every 11 years. Our Sun is a huge ball of electrically-charged hot gas. This charged gas moves, generating a
powerful magnetic field. The Sun's magnetic field goes through a cycle called the solar cycle.
Source: https://spaceplace.nasa.gov/solar-activity/en/
Sub) Science and Technology, Space, sunspots, photosphere etc,

ForumIAS Offline Guidance Centre


2nd Floor, IAPL House, 19, Pusa Road, Karol Bagh, New Delhi – 110005 | helpdesk@forumias.academy|9821711605
Page 38 of 42

PTS 2021|Simulator Test 3– Solutions|ForumIAS


Q.88)
Ans) c
Exp) Option c is correct.
Statement 1 is incorrect: PMMSY will be implemented over a period of 5 years from FY 2020-21 to FY
2024-25 in all States/Union Territories. Focused attention would be given for fisheries development in
Jammu and Kashmir, Ladakh, Islands, Northeast, and Aspirational Districts through area specific
development plans.
Statement 2 is correct: Activities like Mariculture, Seaweed cultivation and Ornamental Fisheries having
potential to generate huge employment will be promoted by the scheme.
Statement 3 is correct: The PMMSY will be implemented as an umbrella scheme with two separate
Components namely (a) Central Sector Scheme (CS) and (b) Centrally Sponsored Scheme (CSS). For
Central Sector Scheme (CS) entire project/unit cost will be borne by the Central government (i.e. 100%
central funding). For Centrally Sponsored Scheme (CSS), North Eastern & Himalayan States: 90% Central
share and 10% State share, Other States: 60% Central share and 40% State share, Union Territories (with
legislature and without legislature): 100% Central share
Source: https://pib.gov.in/Pressreleaseshare.aspx?PRID=1625314
https://pib.gov.in/PressReleasePage.aspx?PRID=1652573
Sub) Economics, Schemes, Aatmanirbhar Bharat Rozgar Yojana

Q.89)
Ans) b
Exp) Option b is correct.
Option b is correct. Personal disposable Income is the part of National Income which is received by the
households excluding taxes and fines.
Personal Disposable Income (PDI ) ≡ PI – Personal tax payments – Non-tax payments (such as fines etc)
Thus, Personal Disposable Income is the part of aggregate income which belong to the households. They
may decide to consume a part of it, and save the rest.
Option a is incorrect. Personal Income is the part of National Income which is received by the households.
Option c is incorrect. National disposable income is the maximum amount of goods and services the
domestic economy has at its disposal.
Option d is incorrect. Gross domestic product refers to the monetary value of all finished goods and services
made within a country during a specific period. Source: https://blog.forumias.com/eco-101-basics-of-
indias-national-income-gdp-gnp-nnp-gva-etc-explained/
Sub) Economics, Schemes, Disposable income

Q.90)
Ans) c
Exp) Option c is correct.
Statement 1 is correct. The Office of the Chief Justice of India (CJI) is declared as a ‘public authority’ under
the Right to Information (RTI) Act by a five-judge Constitution Bench in a 2019 judgement. The Court also
held that the personal information of judges should only be divulged under RTI if such disclosure served the
larger public interest. The disclosure of personal information was discretionary under the RTI Act. The
statute has given the discretion to the Public Information Officer (PIO).
Statement 2 is correct. The Supreme Court collegium’s meetings do not come under the Right to
Information (RTI) Act. The Supreme Court has refused to divulge information under RTI about the
collegium’s confidential communications to protect the independence of the judiciary.

ForumIAS Offline Guidance Centre


2nd Floor, IAPL House, 19, Pusa Road, Karol Bagh, New Delhi – 110005 | helpdesk@forumias.academy|9821711605
Page 39 of 42

PTS 2021|Simulator Test 3– Solutions|ForumIAS


Source: https://www.livemint.com/news/india/cji-falls-under-the-ambit-or-rti-act-rules-supreme-court-
11573635935500.html
Sub) Polity, Indian Constitution, RTI and Judiciary relation

Q.91)
Ans) c
Exp) Option c is correct.
Air temperatures across the Arctic have been increasing at a rate that is about twice the global average . This
phenomenon is called Arctic amplification. Reasons for this could be ozone hole due to ozone depleting
substances, due to more open seas in winters, the growth of sea ice is delayed, leading to disproportionate
loss of sea ice occurring during summer etc.
Source: https://www.downtoearth.org.in/news/climate-change/arctic-sea-ice-is-melting-faster-than-
ever-study-72792
Sub) Environment, Biodiversity, Arctic Amplification

Q.92)
Ans) b
Exp) Option b is correct.
Statement a is incorrect. Additional Grant is granted when a need has arisen during the current financial
year for additional expenditure upon some new service not contemplated in the budget for that year.
Statement b is correct. Supplementary Grant is granted when the amount authorised by the Parliament
through the appropriation act for a particular service for the current financial year is found to be
insufficient for that year.
Statement c is incorrect. Excess grant is granted when money has been spent on any service during a
financial year in excess of the amount granted for that service in the budget for that year. It is voted by the
Lok Sabha after the financial year. Before the demands for excess grants are submitted to the Lok Sabha for
voting, they must be approved by the Public Accounts Committee of Parliament.
Statement d is incorrect. Exceptional Grant is granted for a special purpose and forms no part of the current
service of any financial year.
Source: Indian Polity by M. Laxmikanth. Chapter 14, Centre-State relation
Sub) Polity, Indian Constitution, Article 115 - Financial Grants - Fiscal federalism

Q.93)
Ans) d
Exp) Option d is correct.
Relevance: Bengal in news due to elections.
Parallels between Atmanirbhar Bharat and Swadeshi Movement.
Important static topic asked frequently.
Statement 1 is correct: Apart from helping in starting indigenous business enterprises, Subramaniam
Bharati also wrote a poem dedicated to the struggle against British rule titled Swadesh Geetham. V.O.
Chidambaram Pillai, Subramaniam Bharti and Subramaniam Siva were a few neo nationalist leaders of the
branch of Swadeshi movement in Tamil Nadu.
Statement 2 is correct: Swadeshi spirit found expression in development of indigenous industries like
Swadeshi Steam Navigation Company by V.O. Chidambaram Pillai. The navigation company gave a
challenge to the British Indian Steam Navigation Company. Other indigenous industries like Swadeshi
textile mills, soap, match factories, tanneries etc. also came up.

ForumIAS Offline Guidance Centre


2nd Floor, IAPL House, 19, Pusa Road, Karol Bagh, New Delhi – 110005 | helpdesk@forumias.academy|9821711605
Page 40 of 42

PTS 2021|Simulator Test 3– Solutions|ForumIAS


Statement 3 is correct: Traditional folk theatre forms such as jatras were extensively used in disseminating
the Swadeshi message in an intelligible form to vast sections of the people, many of whom were being
introduced to modern political ideas for the first time. Knowledge Base:
The Swadeshi Movement had its genesis in the anti-partition movement which was started to oppose the
British decision to partition Bengal. On August 7, 1905, with the passage of the Boycott Resolution in a
massive meeting held in the Calcutta Townhall, the formal proclamation of the Swadeshi Movement was
made. The militant nationalists tried to transform the anti-partition and Swadeshi Movement into a mass
struggle and gave the slogan of India’s independence from foreign rule.
New Forms of struggle:
1) Boycott of foreign goods - cloth, salt, sugar, etc
2) Public meetings and processions.
3) Corps of Volunteers or ‘Samitis’ - e.g., Swadesh Bandhab Samiti
4) Imaginative use of traditional festivals and melas
5) Self-reliance/ Atma Shakti (national education)
6) Swadeshi/ Indigenous enterprises.
7) Cultural expressions against colonial rule (Bharat Mata painting, songs - Amar sonar bangla, poems, etc.)
Source: A Brief History of Modern India - Spectrum, Ch - 12
Sub) History, Modern History, Swadeshi Movement - aatma nirbhar

Q.94)
Ans) a
Exp) option a is correct.
Statement 1 is correct, FPOs will be developed by specialist Cluster Based Business Organizations (CBBOs)
engaged by implementing agencies. NAFED, through CBBOs and the Indian Society of Agribusiness
Professionals (ISAP) has initiated the formation and promotion of FPOs of beekeepers and honey collectors
in 5 states of India. These 5 locations are East Champaran (Bihar), Morena (Madhya Pradesh), Bharatpur
(Rajasthan), Mathura (Uttar Pradesh) and Sunderbans (West Bengal).
Statement 2 is incorrect, Initially, there will be three implementing agencies to form and promote FPOs,
namely Small Farmers Agri-business Consortium (SFAC), National Cooperative Development
Corporation (NCDC) and National Bank for Agriculture and Rural Development (NABARD).
NAFED has been appointed as the 4th national implementing agency.
Source: https://pib.gov.in/PressReleasePage.aspx?PRID=1676047
Sub) Economics, Indian Economy, Honey FPO Programme

Q.95)
Ans) c
Exp) Option c is correct.
Avian botulism killed over 18,000 birds in and around Rajasthan’s Sambhar lake, the Indian Veterinary
Research Institute (IVRI), Bareilly, said in a report.
Botulinum is a natural toxin produced by a bacteria known as Clostridium botulin. It produces the toxin
when it starts reproducing.
The bacteria is commonly found in the soil, river, and sea water. There are around eight types — A, B, C1,
C2, D, E, F, and G — of botulinum toxin and they are distinguishable when diagnosed. But all types of
toxins attack the neurons, which leads to muscle paralysis, states the study.
Source: https://www.downtoearth.org.in/news/climate-change/avian-botulism-killed-18-000-birds-at-
sambhar-govt-report-67866

ForumIAS Offline Guidance Centre


2nd Floor, IAPL House, 19, Pusa Road, Karol Bagh, New Delhi – 110005 | helpdesk@forumias.academy|9821711605
Page 41 of 42

PTS 2021|Simulator Test 3– Solutions|ForumIAS


Sub) Environment, Biodiversity, Migratory Birds in Sambhar Lake

Q.96)
Ans) c
Exp) Option c is correct.
Statement 1 is incorrect. The CEC is not appointed by the governor of the state. The Election Commission
of India designates or nominates an officer of the Government of State/Union Territory as the Chief
Electoral Officer of that State or Union Territory as per provisions in section 13 A of the Representation of
the People Act, 1950.
Statement 2 is correct. The Chief Electoral Officer has to supervise the preparation, revision and correction
of electoral rolls in State subject to the superintendence, direction and control of the Election Commission
of India. He also has to supervise the conduct of all elections to Lok Sabha, Rajya Sabha, Legislative
Assembly and Legislative Council in the State subject to the superintendence, direction and control of the
Election Commission of India.
Statement 3 is correct. The Chief Electoral Officer is also empowered to hear appeals against the orders
passed by the Electoral Registration Officers under Section 24 of the Representation of the People Act,
1950. The Election Commission of India in consultation with State Government designates an officer of
Government as Electoral Registration Officer for each Assembly/Council Constituency. The general duty
of the Electoral Registration Officer is to prepare the defect-free Electoral Roll for his constituency.
Source: Indian Polity by M. Laxmikanth. Chapter 38. Election Commission.
Sub) Polity, Indian Constitution, Chief electoral officer - elections going on

Q.97)
Ans) a
Exp) Option a is correct.
Statement 1 is correct. The Constitution authorises the Parliament to form new states or alter the areas,
boundaries or names of the existing states without their consent. Thus, the Parliament can redraw the
political map of India according to its will. Hence, the territorial integrity or continued existence of any state
is not guaranteed by the Constitution.
Statement 2 is correct. An amendment to the Constitution is needed to establish a new state, though it is not
a Constitutional Amendment under Article 368. A creation of a new state will require changes in the First
and Fourth Schedules of the Constitution.
Statement 3 is incorrect. The Supreme Court in 1969 has ruled that the settlement of a boundary dispute
between India and another country does not require a constitutional amendment. It can be done by
executive action. On the other hand, Indian territory can be ceded to a foreign state only by amending the
Constitution under Article 368.
Statement 4 is incorrect. Union Territories with legislative assemblies need not be consulted before making
changes in their boundaries. In case of any union territory, no reference need be made to the concerned
legislature to ascertain its views and the Parliament can itself take any action as it deems fit.
Source: Indian polity by M. Laxmikanth. Chapter 5. The Union and its territory
Sub) Polity, Indian Constitution, Article 3 - Formation of new state – static

Q.98)
Ans) c
Exp) Option c is correct.

ForumIAS Offline Guidance Centre


2nd Floor, IAPL House, 19, Pusa Road, Karol Bagh, New Delhi – 110005 | helpdesk@forumias.academy|9821711605
Page 42 of 42

PTS 2021|Simulator Test 3– Solutions|ForumIAS


Fault is a fracture in the rock-bed found at layers below surface of the earth. The region around the fault is
weak and can lead to the movement and displacement, thus frequent earthquakes can be observed. Malda
Fault was observed at a place called Malda in West Bengal. This facture was due to the formation of fold
mountains (Himalayas). This fault separates Chota Nagpur Plateau and North-Eastern Himalayan
mountains. This Malda gap is also called the Garo-Rajmahal gap.
Option a is incorrect – Kachchh mainland and Rann of Kachchh are separated by the Kachchh Mainland
fault (KMF).
Option b is incorrect – Fault that runs parallel to Satpura and Vindhya ranges forms a rift valley in which
Narmada River flows.
Option d is incorrect – The Palakkad gap or Palghat Gap forms a major break in the Western Ghats
Mountain range, in south-western India. Located between the Nilgiri Hills to the north and the Anaimalai
Hills to the south, it serves as a major communication route between Kerala and Tamil Nadu.
Source: NCERT Class XI – India: Physical Environment, Chapter 2, page 8
Sub) Geography, Indian Physical Geography, Malda Fault

Q.99)
Ans) c
Exp) Option c is correct.
The fundamental right to freedom of conscience is protected under Article 25 of the Constitution. Freedom
of conscience is inner freedom of an individual to mould his relation with God or Creatures in whatever
way he desires. Article 25 also protects an individual’s right to freely profess, practice and propagate
religion.
The Supreme Court in its various judgements has declared Right to privacy, right to health and right to
speedy trial as part of Article 21.
Source: The Constitution of India. PM Bakshi. Page 63
Sub) Polity, Indian Constitution, Rights interpreted from article 21 (e.g. Right to speedy trial)

Q.100)
Ans) b
Exp) Option b is correct.
Statement 1 is incorrect. The 69th Constitutional Amendment Act of 1991 provided a
special status to the National Capital Territory (NCT) of Delhi under Article 239AA of the Constitution.
The legislative assembly can make laws on all the matters of the State List and the Concurrent List.
However, the legislative assembly cannot make laws on public order, police and land.
Statement 2 is correct. The chief minister of the National Capital Territory (NCT) of Delhi is appointed by
the President. The other ministers are appointed by the president on the advice of the chief minister. All the
ministers hold office during the pleasure of the president as they are appointed by him/her.
Statement 3 is incorrect. The lieutenant governor of the National Capital Territory (NCT) of Delhi is
empowered to promulgate ordinances when the assembly is not in session. But no ordinance can be
promulgated or withdrawn by the lieutenant governor without the prior permission of the President. An
ordinance has the same force as an act of the assembly.
Source: https://www.barandbench.com/news/litigation/govt-of-national-capital-territory-of-delhi-
amendment-act-2021-comes-into-force-govt-of-delhi-will-mean-lieutenant-governor
Sub) Polity, Indian Constitution, Status of NCR - SC judgement.

ForumIAS Offline Guidance Centre


2nd Floor, IAPL House, 19, Pusa Road, Karol Bagh, New Delhi – 110005 | helpdesk@forumias.academy|9821711605

You might also like